nclex Exam 3

अब Quizwiz के साथ अपने होमवर्क और परीक्षाओं को एस करें!

The nurse is assessing a 2-year-old who has a blistered sunburn across the back and shoulders. Which of the following parent statements indicates an appropriate understanding of care for sunburn? Select all that apply. 1. "I am allowing my child to play outdoors only very early in the morning and late in the evening since the sunburn." 2. "I am encouraging extra fluids since my child got sunburned." 3. "I have been giving my child acetaminophen to help relieve the pain." 4. "I have been placing cool, wet washcloths on my child's back." 5. "I have rubbed hydrocortisone cream on the area to help reduce inflammation and promote healing."

1. "I am allowing my child to play outdoors only very early in the morning and late in the evening since the sunburn." 2. "I am encouraging extra fluids since my child got sunburned." 3. "I have been giving my child acetaminophen to help relieve the pain." 4. "I have been placing cool, wet washcloths on my child's back." Sunburn is a painful inflammatory skin reaction resulting from overexposure to ultraviolet radiation (eg, natural sunlight, tanning beds). Sunburns may be classified as superficial (ie, red, painful) or partial-thickness (ie, blistering, weeping) burns. Severe sunburns may cause systemic symptoms such as fever, chills, nausea, and headache. Sunburns increase insensible fluid loss and place the client at an increased risk for dehydration. Sunburn prevention is important because sunburn may cause permanent skin damage and increases the risk of skin cancers. However, when minor sunburns occur, symptom management includes: Protecting the burned area from further sun exposure (eg, avoid going outside during midday when the sun's rays are hottest) (Option 1) Promoting increased fluid intake to avoid dehydration (Option 2) Providing pain relief with over-the-counter analgesics such as ibuprofen or acetaminophen (Option 3) Reducing inflammation and pain by taking tepid baths; using cool compresses; and applying soothing, protective lotions or gels (eg, aloe vera, calamine) to the sunburned area (Option 4) (Option 5) Corticosteroid creams (eg, hydrocortisone) have not been shown to reduce symptom severity or healing times. Some preparations can be drying to the skin, which may exacerbate symptoms of sunburn. Educational objective:Sunburn is a painful inflammatory skin reaction that results from overexposure to ultraviolet radiation (eg, sunlight). Care for minor sunburn is symptomatic and involves protecting the burn from further sun exposure, increasing fluid intake, taking mild oral analgesics (eg, acetaminophen), and applying cool compresses and soothing lotions. Additional Information Physiological Adaptation NCSBN Client Need

A client with a chronic kidney disease has blood laboratory values as shown in the exhibit. The nurse administers sodium polystyrene sulfonate by mouth per the health care provider's prescription. The nurse evaluates that the therapy is effective when which value is noted on the follow-up results? Click on the exhibit button for additional information. 1. Calcium 7.4 mg/dL (1.85 mmol/L) (3%) 2. Creatinine 4.0 mg/dL (353 µmol/L) (9%) 3. Phosphorus 3.9 mg/dL (1.26 mmol/L) (9%) 4. Potassium 4.9 mEq/L (4.9 mmol/L) (77%)

4. Potassium 4.9 mEq/L (4.9 mmol/L) (77%) The client with kidney disease is at risk for both hyperkalemia (normal potassium 3.5-5.0 mEq/L [3.5-5.0 mmol/L]) and hyperphosphatemia due to reduced glomerular filtration rate. Untreated hyperkalemia may cause life-threatening cardiac arrhythmias. Sodium polystyrene sulfonate (Kayexalate) can be used to treat hyperkalemia. It works in the gastrointestinal tract to trade sodium for potassium, thereby eliminating excess potassium through the stool and reducing the serum potassium level. (Option 1) Serum calcium levels (normal 8.6-10.2 mg/dL [2.15-2.55 mmol/L]) may decrease with diminished renal function due to lower activation of vitamin D and subsequent impaired gut absorption of calcium. Calcium supplements are used to increase the serum calcium level. Sodium polystyrene sulfonate does not affect the serum calcium level. (Option 2) Sodium polystyrene sulfonate does not affect serum creatinine levels. Creatinine levels may decrease after dialysis. (Option 3) Phosphorus is also not filtered with kidney injury and the levels increase in serum (normal 2.4-4.4 mg/dL [0.78-1.42 mmol/L]). Phosphate binders (calcium acetate/carbonate) administered orally eliminate phosphorus through stool. Sodium polystyrene sulfonate does not bind phosphorus. Educational objective:Clients with kidney disease are at risk for hyperkalemia. Sodium polystyrene sulfonate (Kayexalate) works in the gastrointestinal tract to trade sodium for potassium, thereby eliminating excess potassium through the stool and reducing the serum potassium level.

A pediatric client weighing 66 lb is prescribed ibuprofen 5 mg/kg by mouth every 6 hr PRN for fever. It is available as an oral solution of 20 mg/mL. How many milliliters (mL) of ibuprofen should be given to the client per dose? Record your answer using one decimal place.

7.5 Educational objective:To calculate the volume per dose of ibuprofen, the nurse should first identify the prescribed dose (eg, 5 mg/kg/dose) and available medication (eg, 20 mg/mL) and then convert to volume in milliliters per dose (eg, 7.5 mL).

A critical care nurse is caring for a newly admitted client with acute aortic dissection. Which prescription should the nurse prioritize while awaiting surgical revision of the client's aortic dissection? 1. Administer IV labetalol to maintain blood pressure within prescribed parameters (38%) 2. Initiate and maintain strict bed rest and a low-stimulation environment (22%) 3. Monitor bilateral lower extremity peripheral pulse strength (23%) 4. Prepare the client's consent form for surgical repair of the aorta (15%)

Aortic dissection is a tear in the inner lining of the aorta that allows blood to surge between the layers of the arterial wall, separating and weakening the aortic wall. Perfusion to vital organs may become impaired, and the dissection can rapidly progress to life-threatening cardiac tamponade or aortic rupture. Aortic dissection is characterized by acute onset of excruciating, sharp or "ripping" chest pain that radiates to the back. Emergency surgical repair is usually required. Before surgical repair, the priority is decreasing the risk of aortic rupture by maintaining normal pressure in the aorta. Administering IV beta blocker medication (eg, labetalol, metoprolol, propranolol) helps achieve this by lowering the heart rate and blood pressure, which are often elevated with aortic dissection (Option 1). (Option 2) Bed rest and a low-stimulation environment help lower heart rate and blood pressure, but antihypertensive medication is more effective and rapid-acting, making it the highest priority. (Option 3) Assessing peripheral pulses helps monitor for complications of aortic dissection but is not a priority over interventions that reduce the risk of aortic rupture. (Option 4) Informed consent is required before all surgical interventions; however, consent forms can be completed any time prior to surgery and are not a priority over reducing the risk of aortic rupture. Educational objective:Before emergency surgical repair of aortic dissection, the priority is decreasing the risk of aortic rupture by maintaining normal pressure in the aorta. Administering an IV beta blocker helps achieve this by rapidly lowering elevated heart rate and blood pressure.

The nurse is reinforcing education with the parents of a 2-year-old child about diet choices to promote growth. The family observes a strict vegan diet. Which of the following statements by the nurse are appropriate? Select all that apply. 1. "Diets consisting of legumes as the only protein source are sufficient for growth." 2. "It is important to feed your child fortified breads and cereals to help with iron intake." 3. "Preparing meals with vegetables and fruits will ensure sufficient vitamin B12 intake." 4. "Try to pair foods high in iron with foods high in vitamin C to aid iron absorption." 5. "Your child may require calcium and vitamin D supplementation due to lack of dairy intake."

2. "It is important to feed your child fortified breads and cereals to help with iron intake." 4. "Try to pair foods high in iron with foods high in vitamin C to aid iron absorption." 5. "Your child may require calcium and vitamin D supplementation due to lack of dairy intake." With careful monitoring of nutritional intake, a vegan diet (ie, excluding all animal-derived products [eg, meat, dairy, eggs]) can be appropriate for clients in all age groups. Pediatric clients consuming a vegan diet are at increased risk for nutritional deficiencies (eg, protein, calories, calcium, vitamin D, iron, vitamin B12) due to rapid growth and development. Nurses educating clients about preventing nutritional deficiencies in vegan diets should include information about: Iron: Plant sources of iron, which are in smaller quantities and difficult to absorb, should be supplemented with fortified cereals and breads to decrease risk of iron-deficient anemia (Option 2) Vitamin C: Iron absorption is improved when dietary sources of iron and vitamin C are taken together (Option 4) Calcium: Without animal sources of calcium (eg, dairy, eggs, fish), vegan diets require supplementation of calcium and vitamin D for bone health (Option 5) (Option 1) Many plant-based proteins (eg, legumes, grains) do not individually contain all the essential amino acids to support growth and tissue repair; therefore, vegan clients will require further teaching on combinations of protein sources. (Option 3) Fruits and vegetables do not provide vitamin B12. The nurse should educate the parents on the need for multivitamins or fortified grains as quality vitamin B12 sources. Educational objective:Pediatric clients consuming a vegan diet are at risk for dietary deficiencies (eg, iron, protein, calories, vitamin B12, calcium, vitamin D). Parent education about supplementation and adequate food sources of these nutrients is necessary. Additional Information Health Promotion and Maintenance NCSBN Client Need

The nurse is caring for a baby born at 30 weeks gestation and diagnosed with necrotizing enterocolitis. Which nursing action should be implemented? 1. Encourage parents to increase skin-to-skin care (10%) 2. Measure abdominal girth daily (62%) 3. Measure rectal temperature every 3-4 hours (7%) 4. Position client on side and check diaper for stool (18%)

2. Measure abdominal girth daily Necrotizing enterocolitis occurs predominantly in preterm infants secondary to gastrointestinal and immunologic immaturity. On initiation of enteral feeding, bacteria can be introduced into the bowel, where they can proliferate excessively due to compromised immune clearance. This results in inflammation and ischemic necrosis of the intestine. As the disease progresses, the bowel becomes congested and gangrenous with gas collections forming inside the bowel wall. Measuring the client's abdominal girth daily is an important nursing intervention to note any worsening intestinal gas-associated swelling. Clients are made NPO and receive nasogastric suction to decompress the stomach and intestines. Parenteral hydration and nutrition and IV antibiotics are given. (Option 1) Skin-to-skin care (kangaroo care) promotes bonding with a healthy newborn. It is allowed in some instances for premature infants depending on the condition and week of gestation. Skin-to-skin care should be avoided in infants who are not stable as it may cause additional stress. (Option 3) Taking a client's temperature every 3-4 hours is important; however, rectal temperatures should be avoided due to the risk of perforation of the gangrenous, friable colon. (Option 4) To avoid pressure on the abdomen and facilitate observation for a distended abdomen, clients are placed supine and undiapered. Educational objective:Necrotizing enterocolitis is a life-threatening complication in premature infants due to underdeveloped intestine and gut immunity. Frequent abdominal girth measurements are essential to assess for worsening distension. Clients are placed supine and undiapered. Rectal temperatures should be avoided due to the risk of perforation.

The emergency department nurse is obligated to make a report for which situations? Select all that apply. 1. To a client's employer that the client had a car crash while intoxicated 2. To the authorities that an elderly client has suspicious bruising but denies caregiver abuse 3. To the medical examiner of a death following trauma, even if the family refuses autopsy 4. To the spouse of a client that the client has a reportable sexually transmitted disease 5. To the supervisor that an oncoming health care provider has the smell of alcohol on the breath

2. To the authorities that an elderly client has suspicious bruising but denies caregiver abuse 3. To the medical examiner of a death following trauma, even if the family refuses autopsy 5. To the supervisor that an oncoming health care provider has the smell of alcohol on the breath There are several circumstances in which the nurse is legally required to report to appropriate civil authorities: Suspected elder abuse must be reported to the appropriate authorities for investigation. The nurse has a legal obligation to report signs of abuse regardless of clients' ability or willingness to advocate for themselves (Option 2). The nurse should report deaths that meet medical examiner reporting guidelines (eg, suspected to be the result of a crime, trauma, or suicide) to the authorities for investigation. The local medical examiner has the legal authority and obligation to perform an autopsy independent of the family's wishes (Option 3). For the sake of client safety, nurses should immediately report impaired or intoxicated health care workers, regardless of their position (Option 5). Under the Health Insurance Portability and Accountability Act, a client's reason for an emergency department visit cannot be communicated to employers without the client's permission (Option 1). Health authorities must be notified of a reportable sexually transmitted disease regardless of client wishes. Depending on the condition, authorities may report findings to sexual contacts, but it is a violation of client privacy for the nurse to share this information with the client's family or spouse (Option 4). Educational objective:The nurse is required to report an impaired coworker, a suspicious death, and elder abuse to appropriate authorities. The nurse is legally prohibited from sharing health information with employers or family members without the client's permission.

The nurse is presenting an in-service educational session on child abuse and neglect to a class of certified home health aides. Which of the following statements should the nurse include when discussing the characteristics of the typical perpetrator of child abuse? Select all that apply. 1. "Abusers often have a history of growing up in an environment of domestic violence." 2. "Abusers often have a history of substance abuse." 3. "Child abusers always present as being agitated or out of control." 4. "Most child abusers have a sense of low self-esteem." 5. "Teenage parents are particularly vulnerable to abusing their children."

1. "Abusers often have a history of growing up in an environment of domestic violence." 2. "Abusers often have a history of substance abuse. 4. "Most child abusers have a sense of low self-esteem." 5. "Teenage parents are particularly vulnerable to abusing their children." Typical characteristics of child abuse perpetrators include: Unrealistic expectations of the child's performance, behavior, and/or accomplishments; overly critical of the child Confusion between punishment and discipline; having a stern, authoritative approach to discipline Having to cope with ongoing stress and crises such as poverty, violence, illness, lack of social support, and isolation (Option 1) Low self-esteem—a sense of incompetence or unworthiness as a parent (Option 4) A history of substance abuse; use of alcohol or drugs at the time the abuse occurs (Option 2) Punitive treatment and/or abuse as a child Lack of parenting skills, inexperience, minimal knowledge about child care and child development, and young parental age (Option 5) Resentment or rejection of the child Low tolerance for frustration and poor impulse control Attempts to conceal the child's injury or being evasive about an injury; shows little concern about the child's injury (Option 3) Child abusers are not easily identified by appearance; they often appear calm and well in control but may have violent outbursts, typically in private. Educational objective:Child abusers often have a history of growing up in an environment of domestic violence and have a sense of low self-esteem. History of substance abuse is also a risk factor. Teenage parents are particularly vulnerable to abusing their children. Additional Information Psychosocial Integrity NCSBN Client Need

The nurse is caring for a client on the organ donation waiting list for cardiac transplantation. Which teaching topic is most important for the nurse to emphasize at this time? 1. Immunosuppressive therapy as a lifelong commitment (62%) 2. Importance of accurate daily weight monitoring (25%) 3. Importance of periodic endomyocardial biopsies (3%) 4. Maintenance of meticulous surgical incision care (8%)

1. Immunosuppressive therapy as a lifelong commitment Immunosuppressive therapy (eg, mycophenolate, tacrolimus, corticosteroids) is required after organ transplantation to prevent acute and chronic rejection of the organ. This is a lifelong drug regimen for the transplant client, and it has adverse side effects (eg, nephrotoxicity, hepatotoxicity, infection susceptibility). Prior to surgery, the client needs to fully understand the physical, psychological, and financial commitment required. It is important for the nurse at every opportunity to emphasize strict immunosuppressive therapy compliance to prevent acute transplanted organ rejection. (Option 2) Daily weight monitoring is important for identifying signs of heart failure; however, immunosuppressive therapy compliance is the priority. (Option 3) Endomyocardial biopsies are performed regularly, in addition to routine blood tests, to check for signs of rejection. This is important for the client to know; however, it is not the priority over strict immunosuppressive therapy compliance. (Option 4) Surgical incision care and signs of infection are important teaching topics; however, prior to transplantation, it is most important to ensure that the client understands and will comply with lifelong immunosuppressive therapy. Educational objective:Clients on the organ donation waiting list are educated regarding strict compliance with immunosuppressive therapy, which requires a lifelong commitment to prevent acute transplanted organ rejection.

The nurse is caring for a 2-year-old who is receiving a saline enema for treatment of intussusception. Reporting which client finding to the health care provider (HCP) is most important? 1. Passed a normal brown stool (29%) 2. Passed a stool mixed with blood (33%) 3. Stopped crying (9%) 4. Vomited a third time (27%)

1. Passed a normal brown stool (29%) Most cases of intussusception are successfully treated without surgery using hydrostatic (saline) or pneumatic (air) enema. The nurse will monitor for passage of normal brown stool, indicating reduction of intussusception. If this occurs, the HCP should be notified immediately to modify the plan of care and stop all plans for surgery. (Option 2) In intussusception, the stools are mixed with blood and mucus, giving a characteristic "currant jelly" appearance. This is an expected finding. (Option 3) Pain in intussusception is typically intermittent. It occurs every 15-20 minutes, along with screaming and drawing up of the knees. Therefore, if a child stops crying, it may not be due to reduction of intussusception. (Option 4) Intense pain causes spasms of the pyloric muscle that lead to vomiting after each episode. Vomiting tends to resolve once the intussusception is reduced. Educational objective:Reduction of intussusception is often performed with a saline or air enema. The HCP should be notified if there is passage of a normal stool as this indicates reduction of the intussusception. All plans for surgery should be stopped and the plan of care should be modified.

The nurse assesses a client who is receiving methotrexate for rheumatoid arthritis. Which statement by the client is most concerning? 1. "I am nauseated and vomited three times today." (8%) 2. "I drink four large cups of coffee every day." (8%) 3. "I have small, purple spots all over my skin." (54%) 4. "I plan to stop taking birth control today." (28%)

3. "I have small, purple spots all over my skin." Methotrexate is an antirheumatic drug prescribed to treat rheumatoid arthritis. It acts by interfering with folic acid metabolism, which inhibits DNA synthesis and cell reproduction. Adverse effects associated with methotrexate include bone marrow suppression, hepatotoxicity (ie, drug-induced liver injury), and gastrointestinal irritation (eg, nausea, vomiting, diarrhea). Bone marrow suppression is a serious adverse effect that leads to anemia, leukopenia, and thrombocytopenia. Thrombocytopenia (especially platelet count <100,000/mm3 [100 × 109/L]) is characterized by petechiae (ie, small, purple hemorrhagic spots), purpura, and/or other signs of bleeding (eg, melena, hematemesis, bleeding gums) (Option 3). Bone marrow suppression is managed by dose reduction or discontinuation of the medication. (Option 1) Nausea and vomiting are the most common side effects associated with methotrexate. The nurse should notify the health care provider and request a prescription for an antiemetic; however, vomiting is not the priority concern. (Option 2) Some substances decrease the effectiveness of methotrexate (eg, caffeine, folic acid) and should be avoided. (Option 4) Methotrexate is teratogenic, so pregnancy must be prevented. Effective contraceptives must be used throughout treatment and for one ovulatory cycle after completing treatment for women (three months after completion for men). This statement requires follow-up but is not priority as the client has not yet stopped taking birth control. Educational objective:Adverse effects of methotrexate include hepatotoxicity, gastrointestinal irritation, and bone marrow suppression. Bone marrow suppression can lead to anemia, leukopenia, and thrombocytopenia.

The nurse is caring for several clients in a women's health clinic. Based on the data collected, which client's history is most concerning for an increased risk of endometrial cancer? 1. 40-year-old client who has been taking hormonal birth control pills for the past 10 years (8%) 2. 45-year-old client who reports a history of an ectopic pregnancy with a ruptured ovary and two preterm births (10%) 3. 47-year-old client with polycystic ovary syndrome, obesity, and a history of unsuccessful infertility treatments (42%) 4. 60-year-old client who recently had a colposcopy after testing positive for a high-risk type of human papillomavirus (38%)

47-year-old client with polycystic ovary syndrome, obesity, and a history of unsuccessful infertility treatments (42%) Endometrial cancer arises from the inner lining of the uterus and forms after the development of unregulated endometrial overgrowth (ie, hyperplasia). Although typically slow growing, it can metastasize to the myometrium (ie, uterine muscle tissue), cervix, and nearby lymph nodes and eventually beyond the pelvis. Many signs of endometrial cancer are nonspecific (eg, lower back or abdominal pain), but the hallmark symptom is abnormal uterine bleeding (eg, heavy, prolonged, intermenstrual, and/or postmenopausal bleeding). As with many cancers, the client's family and genetic history (eg, BRCA mutation carrier) are significant risk factors; however, prolonged estrogen exposure without adequate progesterone is the greatest risk factor for developing endometrial cancer. Factors increasing estrogen exposure and endometrial cancer risk include: Conditions associated with infrequent or anovulatory menstrual cycles (eg, polycystic ovary syndrome, infertility, late menopause, early menarche) (Option 3) Obesity Tamoxifen (a medication given for breast cancer) (Option 1) Progestin-containing contraceptives (ie, birth control pills) are associated with a decreased endometrial cancer risk because progestins thin the uterine lining, therefore preventing endometrial hyperplasia. (Option 2) Ectopic pregnancy with a ruptured ovary or preterm birth is not associated with endometrial cancer, although never giving birth at term gestation may increase ovarian cancer risk. (Option 4) Infection with a high-risk type of human papillomavirus increases cervical (not endometrial) cancer risk. Educational objective:Endometrial cancer is a slow-growing malignancy that arises from the inner lining of the uterus. Major risk factors include conditions associated with infrequent or anovulatory menstrual cycles (eg, polycystic ovary syndrome, infertility), obesity, and tamoxifen therapy. Additional Information Health Promotion and Maintenance NCSBN Client Need

A client was treated in the emergency department 2 days ago. The nurse makes a follow-up call to say that a culture shows that the client needs an antibiotic. The client's spouse answers the phone, says that the client is at work and doing fine, and that the client does not need the antibiotic. Which is a priority action for the nurse? 1. Call the prescription into the client's pharmacy (2%) 2. Document the spouse's statement in the client's chart (6%) 3. Notify the emergency department physician (8%) 4. Request that the spouse tell the client to call back (81%)

4. Request that the spouse tell the client to call back The spouse does not have the authority to refuse the required medication for the client as the client is competent and has decision-making capacity. An informed refusal includes knowing the risks and benefits of the decision, including the potential of latent infection/damage in this case. If the client does not call back, the typical facility policy is to try to reach the client by phone 3 times, then by certified letter, and (depending on the seriousness of the result) then sending the police to contact the client. (Option 1) The prescription can be called into the pharmacy, but there is no guarantee that the client will pick it up and take it in light of the spouse's response. Speaking to the client is the priority. (Option 2) The statement and attempts for contact should be documented, but the first priority is client care. (Option 3) The emergency department physician should be notified of the conversation, but the priority is to speak to the client and explain the importance of the new follow-up treatment. If the client has a primary care provider, the nurse could also communicate with that office to aid follow-up. Educational objective:A competent adult with decision-making capacity can refuse essential treatment; the client's spouse does not have that legal authority. Treatment refusal must include awareness of the risks and benefits. Additional Information Management of Care NCSBN Client Need

The nurse should call the primary health care provider to obtain a new prescription prior to administering which medication to a client with type 1 diabetes mellitus? 1. 10 units regular insulin IV push for blood glucose >250 mg/dL (13.9 mmol/L) (23%) 2. 14 units glargine insulin subcutaneous injection every night at 8:00 PM (12%) 3. 18 units aspart insulin subcutaneous injection 15 minutes before breakfast (14%) 4. 20 units NPH insulin IV push administered every morning at 7:00 AM (49%)

4. 20 units NPH insulin IV push administered every morning at 7:00 AM Subcutaneous injection is the indicated route for NPH insulin administration; it should never be administered via IV push. Regular insulin is the only insulin that can be administered via IV push; this is typically performed only in an acute care facility under close observation by the nurse. (Option 1) Administration of 10 units regular insulin IV push for blood glucose >250 mg/dL (13.9 mmol/L) is appropriate and a new prescription is not required. (Option 2) Administration of 14 units glargine insulin subcutaneous injection every night at 8:00 PM is appropriate and a new prescription is not required. (Option 3) Administration of 18 units aspart insulin subcutaneous injection 15 minutes before breakfast is appropriate and a new prescription is not required. Educational objective:Subcutaneous injection is the indicated route for NPH insulin administration; it should never be administered IV push. Regular insulin is the only insulin that can be administered IV push. Additional Information Pharmacological and Parenteral Therapies NCSBN Client Need

The parent of a 21-day-old male infant reports that the infant is "throwing up a lot." Which assessments should the nurse make to help determine if pyloric stenosis is an issue? Select all that apply. 1. Assess the parent's feeding technique 2. Check for family history of gluten enteropathy 3. Check for history of physiological hyperbilirubinemia 4. Check if the vomiting is projectile 5. Compare current weight to birth weight

1. Assess the parent's feedin 4. Check if the vomiting is projectile 5. Compare current weight to birth weightg technique In pyloric stenosis, there is gradual hypertrophy of the pylorus until symptom onset at age 3-5 weeks. It is common in first-born boys and the etiology is unclear. Pyloric stenosis presents with postprandial projectile vomiting (ejected up to 3 feet) followed by hunger (eg, "hungry vomiter"). This is clearly distinguished from the "wet burps" infants have due to a weak lower esophageal sphincter. The emesis is nonbilious as the obstruction is proximal to the bile duct. Infants have poor weight gain and are often dehydrated (eg, sunken fontanelle, decreased skin turgor, delayed capillary refill). The amount of milk consumed (particularly with bottle feedings) along with the mother's technique (mainly adequate burping) should be assessed to ensure there is no excessive air swallowing or overfeeding as an etiology. (Option 2) At times, formula intolerance or allergy is suspected initially when the infant first starts vomiting. However, celiac disease or gluten enteropathy is related to intolerance to gluten, a protein in barley, rye, oats, and wheat (BROW). Clients with celiac disease cannot eat these foods. A 3-week-old infant would only consume milk; this history would not be a factor at this time. (Option 3) Physiological hyperbilirubinemia occurs due to the newborn's immature liver that is unable to metabolize hemoglobin byproducts. This is a "normal" finding that is unrelated to pyloric stenosis. Educational objective:Pyloric stenosis is a hypertrophy of the pyloric sphincter that eventually causes complete obstruction. Classic signs include projectile nonbilious vomiting, an olive-shaped right upper quadrant mass, weight loss, dehydration, and/or electrolyte imbalance (metabolic alkalosis).

A nurse is assessing a client with type 2 diabetes mellitus who was recently started on pioglitazone. Which client data obtained by the nurse is most important to bring to the attention of the health care provider? 1. Bilateral pitting edema in ankles (22%) 2. Blood pressure is 140/88 mm Hg (3%) 3. Most recent HbA1c is 6.7% (9%) 4. Retinal photocoagulation in right eye (64%)

1. Bilateral pitting edema in ankles Thiazolidinediones (rosiglitazone [Avandia] and pioglitazone [Actos]) are used to treat type 2 diabetes mellitus. These agents improve insulin sensitivity but do not release excess insulin, leading to a low risk for hypoglycemia (similar to metformin). These drugs can worsen heart failure by causing fluid retention and increase the risk of bladder cancer. Heart failure or volume overload is a contraindication to thiazolidinedione use. These medications also increase the risk of cardiovascular events such as myocardial infarction.

The nurse is providing care to a 9-year-old client who is awaiting surgery. Which intervention is developmentally appropriate for this client's plan of care? 1. Discuss the procedure with the client using simple diagrams with correct anatomical terminology (62%) 2. Explore the client's perception of how the surgery will positively affect their future (14%) 3. Focus primarily on the client's feelings and concerns regarding surgical scar appearance (13%) 4. Provide initial education about the procedure to the client immediately before it is performed (9%)

1. Discuss the procedure with the client using simple diagrams with correct anatomical termino Planning care during a pediatric hospitalization requires the nurse to consider the child's stage of psychosocial and cognitive development. For the school-age child (age 6-12 years), developing a sense of industry (ie, confidence in skills and abilities) is a primary psychosocial need, and cognitive development is marked by concrete thinking (ie, based on actual objects or activities). During preprocedural education, the nurse should foster a sense of industry by involving the child in discussions about the procedure, interacting with the child directly, and using correct anatomical terminology. In addition, the use of simple diagrams helps to meet the child's need for concrete learning (Option 1). (Option 2) A school-age child has a concrete way of thinking and may not think abstractly about the future until adolescence. (Option 3) Adolescents are more aware of body image than school-age children. Exploring feelings and concerns regarding the appearance of a surgical scar is not a primary focus. (Option 4) Toddlers (age 1-3 years) should receive initial teaching about a procedure immediately before it occurs due to a limited concept of time. School-age children can be prepared in advance of a procedure (eg, a day before), which gives them time to process. Educational objective:During preprocedural education, the nurse should use developmentally appropriate methods of teaching. Using simple diagrams with correct anatomical terminology appropriately meets the psychosocial (ie, sense of industry) and cognitive needs (ie, concrete thinking) of school-aged children. Additional Information Health Promotion and Maintenance NCSBN Client Needlogy

A parent calls the nurse telehealth triage line with concerns about an allergic reaction to something a child ate. Which symptoms should the nurse instruct the parent to assess for to determine if the child is having an anaphylactic reaction? Select all that apply. 1. Dyspnea 2. Fever 3. Lightheadedness 4. Skin rash (hives) 5. Wheezing

1.Dyspnea 3. Lightheadedness 4. Skin rash (hives) 5. Wheezing The nurse should instruct the parent to first assess for signs of swelling of the mouth, tongue, lips, and upper airway. The child will have wheezing and difficulty breathing next, followed soon by cardiovascular symptoms. These include lightheadedness due to hypotension, loss of consciousness, and cardiovascular collapse. An anaphylactic reaction is life-threatening and requires rapid assessment and intervention. (Option 2) Fever is not a symptom of an anaphylactic reaction that would be included in the rapid assessment. Educational objective:Anaphylaxis is a medical emergency requiring rapid assessment and intervention. Symptoms of an anaphylactic reaction include signs of respiratory compromise (eg, oral and airway swelling, stridor, wheezing, chest tightness) and shock (eg, dizziness, loss of consciousness).

After a daily weigh-in, a client with anorexia nervosa realizes a 2-lb weight gain. The client says to the nurse in a distressed voice, "This is terrible. I'm so fat." What is the best response by the nurse? 1. "But you look so thin." (0%) 2. "I don't see you that way; you are making progress toward a healthy weight." (86%) 3. "If you continue to gain weight at this rate, you will be able to go home soon." (10%) 4. "You are not fat; it's all in your imagination." (2%)

2. "I don't see you that way; you are making progress toward a healthy weight." ( A nursing diagnosis associated with anorexia nervosa is disturbed body image/low self esteem. There is often a large disparity between actual weight and the client's perceived weight. Clients with anorexia nervosa think of themselves as overweight and fat. The nursing care plan should include helping the client develop a realistic perception of weight and body image. The nurse can confront the client about the misinterpretation of body weight by presenting reality without challenging the client's illogical thinking. The client's weight should be discussed in the context of overall health. The nurse also needs to be aware of his/her own reaction to the client's behaviors and statement. It is not uncommon for caregivers and care providers to feel frustrated or even angry when caring for a client with an eating disorder. The nurse must maintain a neutral attitude and approach, avoiding arguing or disagreeing with the client's statements. (Option 1) This response is judgmental, reinforces the idea of "thinness," and does not help the client develop a more realistic body image. (Option 3) Establishing a goal weight is part of the nursing care plan for the client with anorexia nervosa; clients are usually not discharged from inpatient treatment until goal weight is achieved. However, this response does not address the client's misperception of body weight. (Option 4) This response dismisses the client's concern and does not present the reality of the situation. Educational objective:Clients with anorexia nervosa have disturbed body image and see themselves as being fat or overweight even when they are severely underweight or even at a normal body weight. The nurse can help the client develop a more realistic self image by presenting the situation realistically and discussing weight in terms of the client's health. Additional Information Psychosocial Integrity NCSBN Client Need

The nurse is performing telephone triage with a client at 38 weeks gestation who thinks she may be in labor. Which questions would help the nurse determine whether the client is in labor? Select all that apply. 1. "Do you feel like the contractions are getting stronger?" 2. "Does anything you do make the pain better?" 3. "Have you lost your mucous plug?" 4. "How frequent are the contractions?" 5. "Where do you feel the contraction pain most?"

1. "Do you feel like the contractions are getting stronger?" 2. "Does anything you do make the pain better?" 4. "How frequent are the contractions?" 5. "Where do you feel the contraction pain most?" True labor is defined as contractions that cause progressive cervical change over time. Probable signs of labor are identified by assessing the timing and intensity of contractions, the success of comfort measures in relieving the pain, and the location of the pain (Options 2 and 5). Consistent, intense contractions that get stronger and closer together (more frequent over time) and are associated with lower back discomfort that radiates to the abdomen are indicative of true labor (Options 1 and 4). If a woman is experiencing Braxton Hicks contractions (ie, "false labor"), the nurse should provide encouragement and education about signs of labor and suggest comfort measures. Comfort measures relieve maternal anxiety, increase coping, and encourage normal progression of labor. The nurse may suggest walking, taking a warm bath, resting in a lateral position, having a snack, staying hydrated, and voiding often. (Option 3) During pregnancy, a collection of secretions forms a "mucous plug" in the cervical canal, acting as a protective barrier. Although the client may notice expulsion of the mucous plug in the days preceding labor, it is not necessarily a sign of labor. Educational objective:True labor is defined as contractions that cause progressive cervical change over time. Consistent, intense contractions that get stronger and closer together and are associated with lower back discomfort that radiates to the abdomen are indicative of probable labor. Additional Information Health Promotion and Maintenance NCSBN Client Need

A nurse is teaching home management to a client newly diagnosed with severe psoriasis. Which client statement indicates that further teaching is needed? 1. "Exposure to sunlight will worsen my psoriasis." (31%) 2. "I should avoid drinking alcohol." (8%) 3. "I should use moisturizing creams frequently." (51%) 4. "Stress can worsen psoriasis." (8%)

1. "Exposure to sunlight will worsen my psoriasis." (31% Psoriasis is a chronic autoimmune disease that causes a rapid turnover of epidermal cells. Characteristic silver plaques on reddened skin may be found bilaterally on the elbows, knees, scalp, lower back, and/or buttocks. The goal of therapy is to slow epidermal turnover, heal lesions, and control exacerbations. There is no cure for psoriasis; disease management includes avoidance of triggers (eg, stress, trauma, infection), topical therapy (eg, corticosteroids, moisturizers), phototherapy (eg, ultraviolet light), and systemic medications, including cytotoxic (eg, methotrexate) and biologic (eg, infliximab) agents (Options 3 and 4). The client should avoid alcohol as it can worsen psoriasis (Option 2). In addition, the liver, kidneys, and bone marrow are specifically affected by the systemic medications commonly used to control psoriasis. (Option 1) Exposure to ultraviolet light (eg, phototherapy, sunlight) can help slow epidermal turnover and decrease exacerbations; however, there is a greater long-term risk of skin cancer. Therefore, frequent skin examinations by a health care provider are important. Educational objective:Psoriasis is a chronic autoimmune condition characterized by exacerbations of silver plaques on reddened skin. Although there is no cure, management includes topical and systemic medications, phototherapy, and avoidance of triggers.

The nurse in a psychiatric clinic is evaluating the client's response to treatment for somatic symptom disorder with cardiac manifestations. Which client statement indicates a need for further treatment? 1. "I am looking for another heart specialist to evaluate my symptoms." (72%) 2. "I asked my spouse for support while I deal with my mother's death." (14%) 3. "I have started carrying a sketchbook to draw in when I am stressed." (2%) 4. "I journal daily about my stress level and any heart-related symptoms." (9%)

1. "I am looking for another heart specialist to evaluate my symptoms. Somatic symptom disorder (SSD) is a psychological disorder in response to stress that results in symptoms of physical disorders (eg, chest pain, syncope) for which there is no identifiable medical source (eg, myocardial infarction, hypotension). Periods of increased stress (eg, work demands, family events) frequently precede the onset, or worsening, of physical symptoms and result in frequent requests for medical attention and treatment. SSD and care-seeking behaviors may then be reinforced and perpetuated by secondary gains (eg, social affirmation, "sick role," avoidance of unpleasant activities). When evaluating clients' responses to treatment for SSD, the nurse should monitor for the following indicators of positive progress: Identification of alternate support systems for stress (eg, spouse, friends) (Option 2) Identification of perceived benefits (ie, secondary gains) of behaviors Use of stress-reducing strategies (eg, drawing, meditating) rather than fixation on symptoms (Option 3) Verbalization of factors causing or worsening symptoms (Option 4) (Option 1) When medical treatment does not support a diagnosis for the physical symptoms, the client may become frustrated and seek the opinion of additional health care providers. This indicates a lack of treatment progress. Educational objective:Somatic symptom disorder (SSD) occurs when psychological stresses manifest as physical symptoms of illness without physiological cause. Treatment has been effective if the client with SSD is able to identify alternate support systems for stress, identify perceived benefits of behaviors, employ stress management strategies, and verbalize factors associated with symptoms. Additional Information Psychosocial Integrity NCSBN Client Need

A client has just returned to the room after having a mammogram. The client is teary and in a shaky voice says to the nurse, "The radiology technician told me that it looks really bad - the tumor in my breast is very large." Which is the best response by the nurse? 1. "I can see that you are very upset. Let's talk about what happened." (93%) 2. "I'll report the technician to the head of the radiology department." (0%) 3. "The technician never should have said that to you." (1%) 4. "Your health care provider will discuss treatment options with you." (4%)

1. "I can see that you are very upset. Let's talk about what happened." (93%) Acknowledging that the client is upset conveys concern and understanding on the part of the nurse and helps establish a therapeutic dialogue. The client can vent feelings and discuss fears because the nurse provides the opportunity to talk about what happened (focusing and listening). This action also establishes interpersonal sensitivity and helps the nurse relate therapeutically to the client. Clients who feel threatened or injured by their medical condition(s) need to feel safe and supported. The nurse is in a unique position to provide the nurturing and caring that clients need as they cope with medical diagnoses and difficult situations. (Option 2) This is not an appropriate response; the proper chain of command would have the nurse report the event to a supervisor. (Option 3) This statement may be true, but it does not facilitate a dialogue about the client's feelings and fears. (Option 4) This response does not address the client's feelings or what happened during the mammogram. Educational objective:Therapeutic communication techniques such as acknowledgement of feelings, focusing, and listening can help establish a dialogue and relationship with a client that is protective, supportive, nurturing, and caring.

A nurse on the antepartum unit is caring for a pregnant client at 30 weeks gestation who was admitted with reports of vaginal bleeding. A diagnosis of placenta previa was confirmed by ultrasound. What should the nurse tell the client to anticipate? Select all that apply. 1. Additional ultrasound around 36 weeks gestation 2. Clearance for sexual activity if bleeding stops 3. Discharge home if bleeding stops and fetal status is reassuring 4. Scheduled cesarean birth before onset of labor 5. Weekly vaginal examinations to assess for cervical change

1. Additional ultrasound around 36 weeks gestation 3. Discharge home if bleeding stops and fetal status is reassuring 4. Scheduled cesarean birth before onset of labor In placenta previa, the placenta is implanted over or very near the cervix. This causes placental blood vessels to be disrupted during cervical dilation and effacement, which may result in massive blood loss and maternal/fetal compromise. Because of the increased risk of hemorrhage if contractions result in cervical change, a cesarean birth is planned for after 36 weeks gestation and prior to the onset of labor (Option 4). A stable client with no active bleeding and reassuring fetal status may be discharged home and managed in an outpatient setting (Option 3). However, the client must be closely monitored and instructed to return to the hospital immediately if bleeding recurs. As pregnancy progresses, the placenta grows in size and can potentially migrate away from the cervical opening, resulting in complete resolution of the previa. Therefore, an additional ultrasound is usually performed around 36 weeks gestation to assess placental location (Option 1). (Options 2 and 5) Clients with placenta previa should be instructed to remain on pelvic rest. Vaginal examinations, douching, and vaginal intercourse are contraindicated due to the risk of disruption of the placental vessels and subsequent hemorrhage. Modified bed rest (ie, decreasing any physical activity that could cause contractions) is also recommended. Educational objective:Clients with placenta previa are at risk for hemorrhage. Vaginal examinations are contraindicated, and pelvic rest is recommended to prevent disruption of placental vessels. A cesarean birth is planned prior to onset of labor. Additional Information Physiological Adaptation NCSBN Client Need

The client has a chest tube for a pneumothorax. While repositioning the client for an x-ray, the technician steps on the tubing and accidently pulls the chest tube out. The client's oxygen saturation drops and the pulse is 132/min; the nurse hears air leaking from the insertion site. What is the nurse's immediate action? 1. Apply an occlusive sterile dressing secured on 3 sides (66%) 2. Apply an occlusive sterile dressing secured on 4 sides (27%) 3. Assess lung sounds (3%) 4. Notify the health care provider (HCP) (2%)

1. Apply an occlusive sterile dressing secured on 3 sides If the chest tube is dislodged from the client and the nurse hears air leaking from the site, the nurse's immediate action should be to apply a sterile occlusive dressing (eg, petroleum jelly dressing) taped on 3 sides. This action permits air to escape on exhalation and inhibits air intake on inspiration. The nurse would then notify the HCP and arrange for the reinsertion of another chest tube (Option 1). (Option 2) A tension pneumothorax develops when air enters the pleural space but cannot escape. Increased intrapleural pressure and excessive accumulation of air can apply pressure to the heart and great vessels and drastically decrease cardiac output. An occlusive dressing taped on 4 sides would prevent the air in the pleural space from escaping on exhalation and would increase the risk for a tension pneumothorax. (Option 3) The nurse would stay with the client, assess lung sounds, and monitor vital signs frequently; however, this is not the immediate action. (Option 4) The nurse would notify the HCP and prepare for reinsertion of a chest tube, but it is not the immediate action. Educational objective:If a chest tube is dislodged from the client and the nurse hears air leaking from the site, the immediate action should be to apply an occlusive sterile dressing taped on 3 sides. This action decreases the risk for a tension pneumothorax by inhibiting air intake on inspiration and allowing air to escape on expiration.

The nurse reviews discharge teaching about residual limb care for a client who had a lower limb amputation. Which of the following instructions should the nurse include? Select all that apply. 1. Assess the residual limb daily for redness or irritation 2. Keep limb socks and elastic wraps clean and dry 3. Lie on your stomach three times a day for 30 minutes 4. Massage the residual limb with lotion each day 5. Wash the residual limb daily with soap and water

1. Assess the residual limb daily for redness or irritation 2. Keep limb socks and elastic wraps clean and dry 3. Lie on your stomach three times a day for 30 minutes 5. Wash the residual limb daily with soap and water Residual limb care following an above-knee amputation (AKA) or a below-knee amputation (BKA) is an important component of rehabilitation and focuses on maintaining skin integrity, controlling pain, preventing infection, and restoring mobility. It is also important for the nurse to consider that the client may experience grief due to disturbed body image. The nurse should include the following residual limb care instructions when discharging a client after an AKA or BKA: Clean the limb by washing it daily with soap and warm water. Thoroughly dry after washing to prevent skin maceration (Option 5). Thoroughly inspect the limb for signs of infection (eg, redness) and areas that may be at risk for infection (eg, irritation, skin breakdown) (Option 1). Keep limb socks, wraps, and appliances/prostheses clean and dry (Option 2). Perform daily range-of-motion exercises to improve muscle strength and mobility. Hip flexion contractures are a common complication during the recovery process. Nurses should teach clients to lie prone several times each day and to avoid sitting in a chair for ≥1 hour (Option 3). (Option 4) Clients should be taught to avoid applying potential irritants (eg, alcohol, lotion, powder) to the residual limb, unless prescribed by the health care provider. This reduces the risk of skin breakdown and infection. Educational objective:Clients who have undergone lower limb amputation should be taught to wash the residual limb daily with warm water and soap; inspect the limb for redness or irritation; keep limb socks, wraps, and appliances clean and dry; and lie prone several times daily to prevent hip contractures. Additional Information Reduction of Risk Potential NCSBN Client Need

The nurse cares for a client who has oral candidiasis. The health care provider has prescribed nystatin oral suspension. Which of the following nursing actions are appropriate? Select all that apply. 1. Assist the client in removing dentures and soaking them in nystatin 2. Inspect the oral mucous membranes thoroughly before administering nystatin 3. Instruct the client to discontinue the medication as soon as symptoms subside 4. Instruct the client to swish the suspension in the mouth for several minutes 5. Shake the bottle of suspension thoroughly before measuring the dose

1. Assist the client in removing dentures and soaking them in nystatin 2. Inspect the oral mucous membranes thoroughly before administering nystatin 4. Instruct the client to swish the suspension in the mouth for several minutes 5. Shake the bottle of suspension thoroughly before measuring the dosen Nystatin is antifungal medication commonly used to treat mucocutaneous candidal infections (ie, oral, intestinal, vaginal, skin). When caring for a client prescribed nystatin, the nurse should: Assist clients with oral candida who wear dentures in removing them and soaking them in nystatin suspension because dentures often become a reservoir for reinfection (Option 1). Assess the appearance of the affected area (eg, oral cavity, skin lesions) frequently throughout nystatin therapy (eg, before administration, during routine assessments) to monitor treatment efficacy and identify potential side effects (eg, mucous membrane irritation) (Option 2). Instruct clients prescribed nystatin liquid suspension for oral thrush to swish the suspension in the mouth for several minutes and then swallow the medication to allow treatment of any esophageal candida (Option 4). Ensure that liquid suspension forms of nystatin are shaken well before being measured for dosing because medication precipitates and causes unequal concentrations within the liquid (Option 5). (Option 3) Clients receiving nystatin should be educated to take the medication as prescribed each day and avoid missing doses; nystatin therapy is continued for at least 48 hours after symptoms subside to prevent recurrence of the infection. Educational objective:Oral nystatin suspension is an antifungal medication used to treat oral thrush caused by candidal infections. Nurses administering nystatin should assist the client in removing and soaking dentures, if present; assess the affected area frequently; educate the client to swish the medication in the mouth before swallowing; and ensure that the suspension is well shaken before dosing.

A client is admitted with a lower urinary tract infection from an obstructing ureteral stone. Which tasks can the registered nurse (RN) delegate to the experienced unlicensed assistive personnel (UAP)? Select all that apply. 1. Assisting the client in completing a health history form 2. Collecting a urine specimen for culture and sensitivity 3. Instructing the client to strain urine when voiding 4. Measuring and documenting urine output 5. Monitoring the color and characteristics of urine output

1. Assisting the client in completing a health history form 4. Measuring and documenting urine output LPN = licensed practical nurse; LVN = licensed vocational nurse; RN = registered nurse;UAP = unlicensed assistive personnel. Measuring intake and output and obtaining a urine specimen for culture and sensitivity are both appropriate duties to delegate to the UAP. Objective measurements (eg, vital signs, intake and output) do not require assessment skills and are therefore appropriate for delegation (Option 4). Nursing actions that require assessment, teaching, evaluation, or clinical judgment must be performed by the RN. Collecting a urine specimen is a routine task with a predictable outcome and is therefore appropriate for delegation to the experienced UAP under the instruction and supervision of the RN (Option 2). The RN should always observe the five rights of delegation by verifying that the UAP have the skills and experience necessary to collect a urine specimen without contamination. If this client had a Foley catheter, specimen collection would be inappropriate for delegation to the UAP. Collecting a specimen from a Foley catheter is considered a sterile procedure as it involves accessing a sterile collection port and risks introducing bacteria into the closed drainage system if done improperly. However, when provided with the appropriate instructions, the UAP and even clients themselves may collect a clean-catch or midstream urine specimen. (Option 1) Conducting a review of the client's complete past medical history is an important responsibility of the RN during the admission assessment. This requires skills and understanding beyond the level of the UAP. (Option 3) Client education about the procedure and rationale for straining urine is the responsibility of the RN. After the RN assesses the knowledge and competency level of the UAP, the UAP can gather supplies, remind the client to strain the urine, and report any abnormal findings to the RN for assessment and validation. (Option 5) Monitoring the color and characteristics of urine output is a qualitative assessment that should be performed by the RN. Educational objective:Collection of routine specimens and measurement of intake and output are appropriate duties for the UAP. Nursing actions that require assessment, teaching, evaluation, or clinical judgment must be performed by the RN.

A nurse is caring for a 3-month-old client with a new tracheostomy. Which findings would indicate a need for suctioning? Select all that apply. 1. Audible gurgling 2. Heart rate 105/min 3. Increased irritability 4. Oxygen saturation 88% 5. Respiratory rate 30/min

1. Audible gurgling 3. Increased irritability 4. Oxygen saturation 88% Artificial airways (eg, tracheostomies, endotracheal tubes) impair the cough mechanism and ciliary function, causing an increase in thick secretions that may occlude the airway. Focused respiratory assessments are critical to determine the need for suctioning and to maintain a patent airway. To decrease the risks associated with the procedure (eg, atelectasis, hypoxemia, trauma, infection), suctioning should be performed only when necessary. Assessment findings that indicate a need for suctioning include: Decreased oxygen saturation (Option 4) Altered mental status (eg, irritability, lethargy) (Option 3) Increased heart rate (normal infant range: 90-160) Increased respiratory rate (normal infant range: 30-60) Increased work of breathing (eg, flared nostrils, use of accessory muscles) Adventitious breath sounds (eg, crackles, wheezes, rhonchi) (Option 1) Pallor, mottled, or cyanotic skin coloring (Options 2 and 5) Respiratory rate of 30/min and heart rate of 105/min are within normal limits for an infant and would not indicate distress or a need for suctioning. Educational objective:Assessment findings that indicate the need to suction a client's tracheostomy or endotracheal tube include decreased oxygen saturation, altered mental status (eg, irritability), increased heart rate or respirations, increased work of breathing, and adventitious breath sounds.

A cardiac catheterization was performed on a client 2 hours ago. The catheter was inserted into the left femoral artery. What signs of potential complications should the nurse report immediately to the health care provider (HCP)? Select all that apply. 1. Bleeding at the catheterization site 2. Client lying down and quietly watching television 3. Client taking only sips of fluids 4. Left foot remarkably cooler than right foot 5. Urine output of 100 mL since the procedure

1. Bleeding at the catheterization site 4. Left foot remarkably cooler than right foot Bleeding at the puncture site indicates that a clot has not formed at the insertion site. This is an arterial bleed as catheterization was done via the femoral artery. Arterial bleeds can lead to hypovolemic shock and death if not treated immediately. Reduced warmth in the lower extremity of the insertion site is a sign of decreased perfusion (lack of oxygenated blood flow) to the extremity and can result in tissue necrosis of the affected area. (Option 2) The client may lie flat for several hours and is encouraged to engage in quiet activities for 24 hours after the procedure to prevent dislodging the clot at the insertion site. (Option 3) Although clients are encouraged to drink fluids to flush dyes out of their system and prevent dehydration, decreased fluid intake would not warrant notifying the HCP. (Option 5) Urine output in this client is above 30 mL/hr and considered to be within the normal range. Educational objective:If not treated immediately, arterial bleeds can lead to hypovolemic shock and death. Reduced warmth in the lower extremity of the catheter insertion site is a sign of decreased perfusion (lack of oxygenated blood flow).

A client who is being evaluated for suspected ectopic pregnancy reports sudden-onset, severe, right lower abdominal pain and dizziness. Which additional assessment findings will the nurse anticipate if the client is experiencing a ruptured ectopic pregnancy? Select all that apply. 1. Blood pressure 82/64 mm Hg 2. Crackles on auscultation 3. Distended jugular veins 4. Pulse 120/min 5. Shoulder pain

1. Blood pressure 82/64 mm Hg 4. Pulse 120/min 5. Shoulde Ectopic pregnancy occurs when a fertilized ovum implants outside the uterine cavity. The majority of ectopic pregnancies occur in the fallopian tubes. Risk factors include recurrent sexually transmitted infections, tubal damage or scarring, intrauterine devices, and previous tubal surgeries (eg, tubal ligation for sterilization). Clinical manifestations are lower-quadrant abdominal pain on one side, mild to moderate vaginal bleeding, and missed or delayed menses. Signs of subsequent hypovolemic (hemorrhagic) shock from ruptured ectopic pregnancy include dizziness, hypotension, and tachycardia. Free intraperitoneal blood pooling under the diaphragm can cause referred shoulder pain. Peritoneal signs (eg, tenderness, rigidity, low-grade fever) may develop subsequently. (Options 2 and 3) Distended jugular veins and lung crackles indicate volume overload. The main risk with ectopic pregnancy is hypovolemic (hemorrhagic) shock. Jugular veins would be flat in hypovolemic shock. Educational objective:The fallopian tubes are the most common site for an ectopic pregnancy. As the ectopic pregnancy grows and expands, rupture may occur, resulting in active bleeding that progresses to life-threatening hypovolemic (hemorrhagic) shock. Signs of ruptured ectopic pregnancy may include severe abdominal pain, dizziness, and referred shoulder pain. Additional Information Physiological Adaptation NCSBN Client Needr pain

The nurse has received report on 4 clients. Which client should the nurse see first? 1. Client admitted this morning with acute pyelonephritis whose IV line is infiltrated (78%) 2. Client scheduled for surgery in 2 hours who has questions about the procedure (9%) 3. Client who had a colostomy yesterday and now has a leaking colostomy bag (6%) 4. Client with a total hip replacement 3 days ago who reports no bowel movement in 2 days (5%)

1. Client admitted this morning with acute pyelonephritis whose IV line is infiltrated Acute pyelonephritis is a severe bacterial infection of the kidney that causes it to swell. It can lead to permanent scarring of the kidney and can be life-threatening. Initial treatment includes vigorous parenteral IV fluids and IV antibiotics. This client's needs are the priority as treatment is dependent on patent IV access. (Option 2) A client scheduled for surgery who has questions about the procedure will need to speak to the health care provider (HCP). The nurse should arrange this as soon as possible as the surgery is scheduled in 2 hours. This client is the second priority. (Option 3) A colostomy is a surgical opening (stoma) in the abdominal wall through which a section of large intestine is brought outside the body, either temporarily or permanently. A colostomy bag is placed over the stoma to collect stool. It is very important that the bag have a good fit and seal around the stoma to prevent skin breakdown due to leaking stool. The colostomy bag can leak if not emptied or cared for properly. This client needs assessment but is not a priority over the client with acute pyelonephritis. (Option 4) The client who had a total hip replacement and reports no bowel movement for 2 days requires assessment. Postoperative pain medications often cause constipation, and client teaching about the importance of adequate fluid and fiber intake needs to be reinforced. The nurse should check the medical record for a prescription for a PRN stool softener, laxative, or enema; if it is not included, the nurse should contact the HCP for further instructions. Educational objective:Clients with acute pyelonephritis require aggressive IV fluids and IV antibiotics to stop progression of the infection and kidney scarring. A patent IV line is the priority.

A nulliparous client asks about being in "real" labor. The nurse should teach that which signs are most indicative of true labor? Select all that apply. 1. Contractions that increase in frequency 2. Contractions that lessen after resting 3. Increased blood-tinged, mucoid vaginal discharge 4. Pain in lower back that moves to lower abdomen 5. Progressive cervical effacement and dilation

1. Contractions that increase in frequency 4. Pain in lower back that moves to lower abdomen 5. Progressive cervical effacement and dilation True labor Contractions Regular intervals; frequency, duration & intensity increase over time False labor Irregular intervals; no increase in frequency, duration, or intensity; may dissipate over time Discomfort Begins in lower back, radiates to abdomen False labor Located in lower abdomen & groin Comfort measures (walking, position changes, hydration) Contractions increase despite comfort measures False labor Contractions may lessen or dissipate with comfort measures Cervical change Increase in cervical dilation & effacement False labor No cervical chages A key indicator of true labor is the progressive effacement and dilation of the cervix (Option 5). Contractions in true labor are regular, and increase in frequency, duration, and intensity (Option 1). The pain may initially start in the lower back and radiate to the abdomen (Option 4). (Option 2) True labor contractions will not lessen or dissipate with comfort measures (eg, rest, position change, hydration) and may intensify with walking or activity. Braxton Hicks contractions are irregular contractions that occur throughout pregnancy, and they do not result in cervical change. Although usually mild, they can be increasingly painful and regular for short periods during the last weeks before birth. However, in contrast with true labor, Braxton Hicks contractions will typically lessen with comfort measures. (Option 3) Mucoid vaginal discharge that is blood-tinged or brownish (eg, bloody show) is a common finding in the days preceding onset of labor; however, it is not a definitive sign that true labor has started. Educational objective:During true labor, contractions increase in frequency, duration, and intensity over time, resulting in progressive dilation and effacement of the cervix. Clients in true labor often experience discomfort in the lower back that radiates to the abdomen with contractions. Contractions associated with true labor do not lessen or dissipate with comfort measures.nge

A client with advanced osteoarthritis is admitted for right total knee arthroplasty. Which characteristic manifestations does the nurse expect to assess in this client? Select all that apply. 1. Crepitus with joint movement 2. Low-grade fever 3. Morning stiffness lasting 10 to 15 minutes 4. Pain exacerbated by weight-bearing activities 5. Positive serum rheumatoid factor

1. Crepitus with joint movement 3. Morning stiffness lasting 10 to 15 minutes 4. Pain exacerbated by weight-bearing activities Osteoarthritis (OA) is a degenerative disorder of the synovial joints (eg, knee, hip, fingers) that causes progressive erosion of the articular (joint) cartilage and bone beneath the cartilage. As the degenerative process continues, bone spurs (osteophytes), calcifications, and ulcerations develop within the joint space, and the "cushion" between the ends of the bones breaks down. Clinical manifestations of OA of the knee include: Pain exacerbated by weight-bearing activities: Results from synovial inflammation, muscle spasm, and nerve irritation (Option 4) Crepitus, a grating noise or sensation with movement that can be heard or palpated: Results from the presence of bone and cartilage fragments that float in the joint space (Option 1) Morning stiffness that subsides within 30 minutes of arising (Option 3) Decreased joint mobility and range of motion Atrophy of the muscles that support the joint (eg, quadriceps, hamstring) due to disuse (Option 2) Low-grade fever develops as part of systemic inflammation. OA is typically a noninflammatory, nonsystemic disorder. Occasional OA inflammation is limited to affected joints. (Option 5) Serum rheumatoid factor is positive in clients with systemic rheumatoid arthritis. No diagnostic laboratory tests or biomarkers exist for OA. Educational objective:Osteoarthritis is a degenerative disorder of the synovial joints that leads to progressive erosion of the articular (joint) cartilage. Clinical manifestations include pain exacerbated by weight-bearing, crepitus, morning stiffness subsiding within 30 minutes, decreased joint mobility and range of motion, and atrophy of supporting muscles.

A client with a history of diverticular disease is being discharged after an episode of acute diverticulitis. Which instructions should be included in the discharge teaching plan to reduce the risk of future episodes? Select all that apply. 1. Drink plenty of fluids 2. Exercise regularly 3. Follow a low-residue diet 4. Include whole grains, fruits, and vegetables in the diet 5. Increase intake of red meat

1. Drink plenty of fluids 2. Exercise regularly 4. Include whole grains, fruits, and vegetables in the diet Diverticular disease of the colon is a condition in which there are sac-like protrusions in the large intestine (diverticula). Diverticulosis is characterized by the presence of these protrusions; the client is asymptomatic and may not even be aware of the condition. Diverticulitis occurs when diverticula become infected and inflamed. Complications of diverticulitis include abscess, fistula formation, intestinal obstruction, peritonitis, and sepsis. Diverticular bleeding occurs when a blood vessel next to one of these pouches bursts; this may cause blood in the stool. The etiology of diverticular disease has been linked to chronic constipation, a major cause of excess intracolonic pressure. Preventing constipation may help reduce the risk of diverticula forming and becoming inflamed. Measures to prevent constipation include a diet high in fiber (whole grains, fruits, vegetables), daily intake of at least 8 glasses of water or other fluids, and exercise. A fiber supplement such as psyllium or bran may be advised. In the past, clients have been taught to avoid consuming seeds, nuts, and popcorn; however, current evidence does not indicate that avoidance of these foods will prevent an episode of diverticulitis. (Option 3) A low-residue diet, which avoids all high-fiber foods, may be used in treating acute diverticulitis. However, after symptoms have resolved, a high-fiber diet is resumed to prevent future episodes. (Option 5) Increased consumption of red meat and other high-fat foods can increase the risk of diverticulitis. Educational objective:Clients with diverticulosis should take measures to prevent constipation (eg, high-fiber diet, increased fluid intake, regular exercise), which may help prevent recurring episodes of acute diverticulitis. Additional Information Physiological Adaptation NCSBN Client Need

A nurse is discussing the fine motor abilities of a 10-month-old infant with the infant's parent. Which are developmentally appropriate skills for an infant of this age? Select all that apply. 1. Grasps a small doll by the arm 2. Stacks 3 wooden blocks 3. Transfers small objects from hand to hand 4. Turns single pages in a book 5. Uses a basic pincer grasp

1. Grasps a small doll by the arm 3. Transfers small objects from hand to hand 5. Uses a basic pincer grasp Fine motor skills of infants develop around the ability to grasp and pick up objects. By 3 months, infants will reflexively grasp a rattle placed in their hand. At 5 months, they are able to voluntarily clasp it with their palm. Around 7 months, infants are able to transfer an object from one hand to the other. By 8-10 months, infants have replaced the palmar grasp with a crude pincer grasp (use of thumb, index, and other fingers) to pick up round oat cereal and other finger foods. By 11 months, this develops into a neat pincer grasp (use of thumb and index finger). (Options 2 and 4) By 12 months, infants may attempt to turn multiple book pages at once, and they also begin attempts to stack 2 blocks. These skills require finer muscle control than is expected of a 10-month-old. Educational objective:Fine motor skills of infants develop around the ability to grasp objects. Voluntary grasping with the palm begins around 5 months, followed by the ability to transfer an object between hands by 7 months and the development of a crude pincer grasp (using the thumb, index, and other fingers) around 8-10 months.

The nurse is preparing a client for a magnetic resonance cholangiopancreatography. Which statements by the client would require the nurse to obtain further assessment data? Select all that apply. 1. "I ate lunch about 4 or 5 hours ago." 2. "I got a rash the last time I had IV contrast." 3. "I had my last period 6 weeks ago." 4. "I have a hearing aid implanted in my ear." 5. "I smoked a cigarette about an hour ago."

2. "I got a rash the last time I had IV contrast." 3. "I had my last period 6 weeks ago." 4. "I have a hearing aid implante Magnetic resonance cholangiopancreatography (MRCP) is a noninvasive diagnostic test used to visualize the biliary, hepatic, and pancreatic ducts via MRI. MRCP uses oral or IV gadolinium (noniodine contrast material) and is a safer, less-invasive alternative to endoscopic retrograde cholangiopancreatography to determine the cause of cholecystitis, cholelithiasis, or biliary obstruction. The nurse must assess for contraindications before the procedure, including the presence of certain metal and/or electrical implants (eg, aneurysm clip, pacemaker, cochlear implant) or any previous allergy or reaction to gadolinium (Option 4). A client with a history of rash following prior IV contrast administration should be assessed to determine the type of contrast that caused the reaction. Although allergies to iodine-based contrast material are more common, the nurse must rule out a gadolinium allergy (Option 2). Pregnancy also is a contraindication for MRCP as gadolinium crosses the placenta and may adversely affect the fetus. Delayed/irregular menses may be a normal variation in some clients; however, delayed menses may indicate pregnancy and should be reported for further investigation prior to MRCP (Option 3). (Option 1) Many clients should be NPO for 4 hours prior to the procedure to allow better visualization of the anatomical features. (Option 5) Smoking does not affect MRI visualization and is not a contraindication. Educational objective:Magnetic resonance cholangiopancreatography uses MRI to visualize the biliary and hepatic ductal system. Contraindications, including pregnancy, the presence of certain metal implants, and an allergy to gadolinium (ie, noniodine contrast agent), should be assessed before the procedure. Additional Information Reduction of Risk Potential NCSBN Client Needd in my ear."

The nurse is assessing a client at 36 weeks gestation during a routine prenatal visit. Which statement by the client should the nurse investigate first? 1. "I am not sleeping as well due to cramps in my calves at night." (29%) 2. "I have noticed less kicking movements as the baby grows bigger." (49%) 3. "Over the last few weeks, I have not been able to wear any of my shoes." (17%) 4. "Sometimes I feel short of breath after walking up a flight of stairs." (3%)

2. "I have noticed less kicking movements as the baby grows bigger." Fetal movement is a sign of fetal health and indicates an intact fetal central nervous system. Fetal movement may occur numerous times per hour during the last trimester of pregnancy, although the client may not perceive every movement. Multiple factors (eg, maternal substance abuse, medications, fasting, fetal sleep) can affect fetal movement. However, fetal movements should not decrease as the fetus increases in size. Decreased fetal movement is a potential warning sign of fetal compromise (ie, impaired oxygenation), which may precede fetal death (Option 2). The nurse prioritizes assessment of client reports of decreased fetal movement to evaluate fetal well-being (eg, nonstress test). (Option 1) Leg cramps commonly occur in the third trimester, especially at night, due to the weight of the gravid uterus applying pressure to nerves affecting calf muscles. Home interventions include stretching legs, massaging calves, and increasing fluid intake. (Option 3) Dependent edema in the lower extremities is common in the third trimester due to decreased venous return (gravid uterus pressure on vena cava), especially with prolonged sitting/standing. This is not a priority over decreased fetal movement. (Option 4) As the uterus rises in the third trimester, the diaphragm is prevented from allowing full lung expansion, causing dyspnea, especially with exertion. Educational objective:Fetal movement is a sign of fetal health and represents an intact fetal central nervous system. The nurse should educate clients that fetal movements do not decrease in the late third trimester and prioritize assessment of clients reporting decreased fetal movement.

The nurse reinforces teaching a client on prescribed dabigatran for chronic atrial fibrillation. Which statement by the client indicates a need for further teaching? 1. "I will call my health care provider if I notice red urine or blood in my stool." (11%) 2. "I will not stop taking dabigatran even if I get a stomachache." (56%) 3. "I will place capsules in my pill box so I will not forget to take a dose." (26%) 4. "I will swallow the capsule whole with a full glass of water." (5%)

2. "I will not stop taking dabigatran even if I get a stomachache Thrombin inhibitors such as dabigatran (Pradaxa) reduce the risk of clot formation and stroke in clients with chronic atrial fibrillation. The nurse should educate the client about implementing bleeding precautions (eg, using a soft-bristle toothbrush, shaving with an electric razor). Dabigatran capsules should be kept in their original container or blister pack until time of use to prevent moisture contamination (Option 3). (Option 1) Red urine or blood in the stool may indicate internal bleeding caused by thrombin inhibitors. The client should report these symptoms to the health care provider. (Option 2) Thrombin inhibitors should only be stopped under the direction of the health care provider. The nurse should educate the client that stopping dabigatran will increase the risk for stroke. Taking the medication with food will not affect how much is absorbed, and food or a full glass of water may prevent gastrointestinal side effects (eg, nausea, indigestion). (Option 4) Thrombin inhibitor capsules should not be crushed or opened as crushing pills increases absorption and risk of bleeding. Educational objective:Thrombin inhibitors such as dabigatran reduce the risk for clots and stroke in clients with chronic atrial fibrillation. The nurse should teach the client to use bleeding precautions and monitor for symptoms of bleeding, swallow capsules whole with a full glass of water, and keep capsules in their original container until time of use.

A client is scheduled for coronary artery bypass surgery in the morning. In the middle of the night, the nurse finds the client wide awake. The client demonstrates symptoms of extreme anxiety and tells the nurse about wanting to refuse the surgery. Which statement by the nurse would be most appropriate? 1. "Please try not to worry, you have an excellent surgeon." (0%) 2. "Tell me about how you feel about your surgery." (93%) 3. "Why are you considering refusing the surgery?" (2%) 4. "You have the right to make your own decisions and can refuse the surgery." (3%)

2. "Tell me about how you feel about your surgery." "Tell me about how you feel about your surgery," is the most appropriate statement to encourage the client to express the source of anxiety. Using an open-ended question enables the client to take control of the conversation and direct it to concerns about the surgery. The nurse can then address the specific concerns identified and provide individualized explanations and support. (Option 1) This statement is nontherapeutic as giving false reassurance minimizes the client's concerns and diminishes trust between the nurse and client. (Option 3) This statement is nontherapeutic and intimidating. Asking "why" and "how" is an ineffective method of gathering information. (Option 4) A client may share a decision with the nurse in an effort to discuss feelings. This statement is nontherapeutic because giving approval of the client's decision does not encourage the client to express concerns about the surgery. Educational objective:Therapeutic conversation techniques (eg, active listening, using open-ended questions) encourage the client to express feelings and ideas and establish an open, trusting relationship with the nurse. Nontherapeutic communication techniques (eg, expressing approval or disapproval, giving advice, asking why) discourage expression of feelings and ideas and close down the conversation between the nurse and client.

Four clients with different skin alterations come to the emergency department. Which client should the nurse advise that the health care provider (HCP) see first? 1. 8-year-old client who uses corticosteroid inhaler and has white patches on the tongue (5%) 2. 50-year-old client who developed a smooth, red, pinpoint rash after taking sulfa (62%) 3. 60-year-old client with pain and crusted blisters along the back (27%) 4. 70-year-old client who has erythema with a small pustule at the hair follicle (4%)

2. 50-year-old client who developed a smooth, red, pinpoint rash after taking sulfa (62% Petechiae (small pinpoint red/purple spots on mucus membrane or skin) and purpura (irregular purplish blotches) can be a sign of blood dyscrasia, including thrombocytopenia due to a severe drug response. This systemic symptom takes priority over a more localized dermatological presentation. (Option 1) Oropharyngeal candidiasis/thrush (moniliasis) is a fungal infection of skin or mucous membranes. It resembles curdled milk and can bleed when removed. The etiology may be due to not rinsing the mouth after steroid inhaler use. It is treated with antifungal suspensions (nystatin) and is nonurgent. (Option 3) Shingles (herpes zoster) is reactivation of dormant varicella virus. The lesions follow the nerve dermatome and can be quite painful. Incidence increases after age 50. Active chickenpox requires airborne and contact precautions, but not the shingles with crusted lesions, especially if the lesions are covered with clothes. It can be contagious to individuals who have not had varicella or who are immunocompromised. However, this is the second priority as this is a localized issue; the nurse can place this client in a private area. (Option 4) Folliculitis is usually due to the presence of staphylococci in moist areas where there is friction. It is most common in the scalp, beard, and extremities in men. It can be treated with medicated soap, topical antibiotics, and warm compresses. The systemic issue in Client 2 is a priority. Educational objective:Petechiae (small circles) and purpura (blotches) are reddish/purple rashes that do not blanch. They indicate systemic blood dyscrasias and are a priority for treatment. Additional Information Physiological Adaptation NCSBN Client Need

A client with end-stage liver disease is admitted for a transplant workup. The client's spouse states that the client has not stopped drinking alcohol and may be unable to quit for 6 months before the transplant. Which is the most appropriate action for the nurse to implement? 1. Ask the transplant team to place a palliative care referral so the client can learn about the option of hospice instead of transplant (1%) 2. Assess the client's motivation to make the necessary self-care changes before and after the transplant (87%) 3. Schedule a meeting to enlist the help of family members in encouraging the client to stay sober until the transplant (2%) 4. Tell the nurse manager that the client may not be an appropriate transplant candidate (9%)

2. Assess the client's motivation to make the necessary self-care changes before and after the transplant The client may not be an appropriate transplant candidate due to his alcohol use. However, additional facts are needed to determine the true situation as the only information obtained came from the client's spouse. The nurse should assess the client's drinking habits and motivation to stop drinking before and after the transplant by speaking with the client directly. In addition, a transplant requires many other self-care regimens. The nurse should be alert for indicators of the client's ability to take prescribed medications, follow dietary restrictions, and attend medical appointments. The information obtained from this assessment should be communicated to the interdisciplinary team members responsible for determining transplant eligibility. (Option 1) Transplant and hospice care are options for the client with end-stage liver disease; palliative specialists can assist with identifying the goals of care and facilitating decision-making. However, this does not directly address the concern of the client's spouse. At this time, further assessment is necessary before appropriate interventions can be planned. (Option 3) At this point, it is unclear whether the client is motivated to quit drinking (the nurse has only heard from the spouse). Therefore, it is premature to plan for this type of intervention. The client will need to stay sober not just prior to the surgery, but for a lifetime if the transplant occurs. Any discussion between the client's family and the medical team should be based on this understanding. (Option 4) If the nurse is concerned about transplant eligibility due to the spouse's statement, then this should be communicated to the client's interdisciplinary team after further assessment. At this point, it is premature to involve the nurse manager. Educational objective:The client with end-stage organ failure must be motivated and able to engage in complex self-care regimens before and after solid organ transplant. Concerns about the client's motivation to engage in necessary self-care requirements (eg, alcohol abstinence) require further assessment. Additional Information Management of Care NCSBN Client Need

A parent calls the after-hours triage nurse about a 3-year-old who is sick with the flu. Which report by the parent would necessitate intervention by the nurse? 1. Acetaminophen being given every 4 hours for fever (17%) 2. Bismuth subsalicylate being used for nausea (52%) 3. Ibuprofen being given every 6 hours for body aches (20%) 4. Popsicles and gelatin desserts being used for hydration (10%)

2. Bismuth subsalicylate being used for nausea The nurse should tell the parent to discontinue the use of bismuth subsalicylate (Pepto-Bismol) as it contains a salicylate (same class as aspirin) and could possibly cause Reye syndrome. Reye syndrome can develop in children with a recent viral illness such as varicella or influenza. It can cause acute encephalopathy and hepatic dysfunction. Children with viral infections should not be given aspirin or products containing salicylates. (Options 1 and 3) Acetaminophen and ibuprofen are being used appropriately. (Option 4) Sufficient fluids are important to maintain hydration in the child with influenza. Water and fluids should be offered frequently; popsicles and gelatin desserts (eg, Jell-O) provide a palatable means of getting children to ingest fluids. Educational objective:The nurse should tell the parent not to administer any product containing aspirin or salicylates to a child with a viral infection (eg, influenza, varicella) to prevent Reye syndrome. Additional Information Pharmacological and Parenteral Therapies NCSBN Client Need

The client has a dislocated shoulder and the nurse is assisting the health care provider with bedside procedural moderate sedation (conscious sedation). During the procedure, the client becomes restless and cries out "Help me!" What action should the nurse take first? 1. Administer midazolam per protocol (20%) 2. Check the client's pulse oximeter (51%) 3. Give more morphine per protocol (7%) 4. Open the airway with head tilt-chin lift (19%)

2. Check the client's pulse oximeter When there is new, sudden onset of restlessness/agitation, the nurse should first think about oxygenation (or blood glucose). The desired level of sedation is level 3 on the Ramsay Sedation Scale, during which the client is drowsy but responds to a voice command. (Option 1) Adequate oxygenation should be established first before administering additional benzodiazepine for sedation. (Option 3) Oxygenation should be assessed before administering additional narcotics for pain. Change in the level of consciousness (restlessness/agitation or lethargy/sedation) can be an indication of excess medication and should be assessed before administering additional drugs. (Option 4) If the client is speaking, the airway is open. Opening the airway would be an initial response if there is new onset of snoring respirations (the tongue falling back due to relaxation and blocking the airway). Normal respirations should be effortless and quiet. Educational objective:When new-onset restlessness occurs during procedural sedation, oxygenation should be considered first before administering additional medications. If the client is snoring, opening the airway should be considered.

The nurse is conducting a follow-up interview with a client who is being treated for depression and suicidal ideation. Which factor best indicates the client is not currently at risk for suicide? 1. Client claims to have more energy and vigor since starting therapy (3%) 2. Client has clear future plans involving personal goals and family milestones (79%) 3. Client has signed a contract promising not to commit suicide (12%) 4. Client reports losing amitriptyline and requests a refill (5%)

2. Client has clear future plans involving personal goals and family milestones ( Clients receiving treatment for depression and suicidal ideation must be carefully monitored for indications of increasing intent. During the client interview, the nurse should assess: Access to psychiatric medications Availability of help during a crisis (eg, counselor, family) Future goals and plans Home and work environment risks Overall affect and level of energy Possible access to weapons Clients who articulate long-term personal goals and family milestones are less likely to commit suicide (Option 2). (Option 1) Clients often feel more energetic after beginning treatment, yet thoughts of suicide may not have fully resolved and the client may now have the energy to follow through with suicide plans. (Option 3) "No-harm/no-suicide" contracts are widely used in clinical practice to support a client's ability to avoid acting on suicidal thoughts. This practice is controversial as there is no evidence that contracts reduce suicide rates. These agreements do not guarantee safety and have no legal credibility. (Option 4) Amitriptyline is a tricyclic antidepressant, an overdose of which is extremely dangerous and likely fatal. Although the nurse may interpret the client's report of having lost the prescription as an attempt to be compliant, the nurse must also be aware that the client may be "stockpiling" medication for a suicide attempt. Educational objective:Nursing care for clients with suicidal ideation includes assessment of home and work environments, access to psychiatric medications, overall affect, and energy level. Clients who articulate long-term personal goals are less likely to commit suicide. Additional Information Psychosocial Integrity NCSBN Client Need

The nurse in the outpatient clinic is reviewing phone messages. Which client should the nurse call back first? 1. Client post kidney transplant who reports white spots in the oral cavity (37%) 2. Client with a history of mitral valve regurgitation who reports fatigue (25%) 3. Client with erythema and purulent drainage at the site of a spider bite (26%) 4. Client with hypertension who reports a cold and nasal congestion (10%)

2. Client with a history of mitral valve regurgitation who reports fatigue Mitral valve regurgitation is the result of a disrupted papillary muscle(s) or ruptured chordae tendineae, allowing a backflow of blood from the left ventricle through the mitral valve into the left atrium. This backflow can lead to dilation of the left atrium, reduced cardiac output, and pulmonary edema. Clients are often asymptomatic but are instructed to report any new symptoms indicative of heart failure (eg, dyspnea, orthopnea, weight gain, cough, fatigue). This client should be assessed first due to possible heart failure, which would require immediate intervention. (Option 1) Kidney transplant recipients are on an immunosuppressant regimen to prevent rejection of the transplanted organ, which can leave them susceptible to infections such as candidiasis (thrush) of the oral cavity. (Option 3) The client with a spider bite is displaying signs and symptoms of infection, and further assessment is required to evaluate for conditions such as cellulitis. This client should be called second. (Option 4) Clients with hypertension who develop sinus or nasal congestion have limited options for symptom relief. Decongestants containing a vasoconstrictor (eg, pseudoephedrine) can exacerbate hypertension. Educational objective:Chronic mitral valve regurgitation is often asymptomatic, but many clients eventually develop heart failure; therefore, early recognition of symptoms is a priority. Mitral regurgitation causes a backflow of blood from the left ventricle to the left atrium, resulting in pulmonary edema (eg, dyspnea, orthopnea) and decreased cardiac output (eg, fatigue). Left atrial enlargement can also result in atrial fibrillation (eg, palpitations).

The nurse receives the following information in the hand-off report. Which client should the nurse assess first? 1. Client with a paralytic ileus following a colon resection who has abdominal distension, no audible bowel sounds, and nausea (14%) 2. Client with alcoholic cirrhosis who has coffee ground nasogastric drainage, blood pressure of 90/60 mm Hg, and pulse of 110/min (69%) 3. Client with bacterial peritonitis following surgery for a ruptured appendix who is receiving IV tobramycin and has a temperature of 101 F (38.3 C) (6%) 4. Client with dysphagia and a sore throat who has a nasogastric tube to administer contrast media for an abdominal CT scan (10%)

2. Client with alcoholic cirrhosis who has coffee ground nasogastric drainage, blood pressure of 90/60 mm Hg, and pulse of 110/min The nurse should first assess the client with alcoholic cirrhosis, as this condition is associated with gastritis, clotting abnormalities (eg, thrombocytopenia, coagulation disorders), and esophageal varices that increase the risk for hemorrhage (coffee ground emesis from oxidized blood). Hypotension and tachycardia in the presence of blood loss can indicate hypovolemia. The nurse should monitor for signs of hemodynamic instability (eg, hypotension, decreased urine output, peripheral vasoconstriction, pallor) and notify the health care provider of any significant changes from baseline as immediate esophagogastroduodenoscopy is necessary to determine the bleeding site. Treatment to stop the bleeding (eg, heat probe, sclerotherapy) may be indicated. (Option 1) A paralytic ileus is a non-mechanical intestinal obstruction that can occur following abdominal surgery. Expected manifestations include absent or hypoactive bowel sounds due to the lack of bowel motility and peristalsis, and abdominal distension and nausea due to the accumulation of gas and fluids in the bowel. (Option 3) Bacterial peritonitis (peritoneal inflammation) involves the entry of bacteria into the peritoneal cavity and is associated with a ruptured appendix. Elevated temperature would be an expected finding. (Option 4) Sore throat discomfort from irritation of the oropharynx is expected in a client with a nasogastric tube. Educational objective:Clients with alcoholic cirrhosis are at increased risk for hemorrhage due to esophageal varices and coagulation disorders. Hypotension and tachycardia in the presence of blood loss can indicate hypovolemia and require immediate assessment.

A client with an asthma exacerbation has been using her albuterol rescue inhaler 10-12 times a day because she cannot take a full breath. What possible side effects of albuterol does the nurse anticipate the client will report? Select all that apply. 1. Constipation 2. Difficulty sleeping 3. Hives with pruritus 4. Palpitations 5. Tremor

2. Difficulty sleeping 4. Palpitations 5. Tremor Albuterol is a short-term beta-adrenergic agonist used as a rescue inhaler to treat reversible airway obstruction associated with asthma. Dosing in an acute asthma exacerbation should not exceed 2-4 puffs every 20 minutes x 3. If albuterol is not effective, an inhaled corticosteroid is indicated to treat the inflammatory component of the disease. Albuterol is a sympathomimetic drug. Expected side effects mimic manifestations related to stimulation of the sympathetic nervous system, and commonly include insomnia, nausea and vomiting, palpitations (from tachycardia), and mild tremor. (Option 1) Constipation is not a common side effect of inhaled beta-agonist drugs. (Option 3) Hives can occur as a sign of an allergic reaction and are not a common anticipated side effect of an inhaled beta-agonist drug. Educational objective:Albuterol is a short-term beta-agonist rescue drug used to control symptoms of airway obstruction and promote bronchodilation. It is a sympathomimetic drug; common expected side effects include insomnia, nausea and vomiting, palpitations (tachycardia), and mild tremor. Additional Information Pharmacological and Parenteral Therapies NCSBN Client Need

A client is newly admitted to the mental health unit with a diagnosis of schizophrenia with persecutory delusions. Which nursing interventions should the nurse include in the client's plan of care with regard to the delusional thinking? Select all that apply. 1. Explore the meaning behind the client's delusions 2. Focus on reality and verbally reinforce it 3. Focus on the client's feelings secondary to the delusions 4. Gently confront the client about the false beliefs 5. Present logical explanations to discredit the delusions

2. Focus on reality and verbally reinforce it 3. Focus on the client's feelings secondary to the delusions Clients with persecutory delusions (paranoid delusions) believe that they are being persecuted or harmed (eg, spied on, cheated, followed, poisoned). Focusing on the client's feelings secondary to the delusion is an example of empathy, one of the most important parts of the therapeutic nurse-client relationship. When nurses attempt to understand clients' feelings and their meaning, clients realize that someone is trying to understand them and the nurse-client relationship grows (Option 3). Focusing on reality and verbally reinforcing it will decrease the time that the client spends thinking about the delusions (Option 2). For example, the nurse may focus on the client's feelings by stating, "I understand that it is frightening to know that someone is trying to poison you." Reality orientation may also be helpful by telling the client, "What you are thinking is part of your disease and not real." (Option 1) Attempting to explore the meaning behind a delusion will encourage the client to focus/think more on this delusion. (Option 4) Confronting the client about the delusion is not therapeutic because arguing will not eradicate the delusion. It also hinders the development of a trusting nurse-client relationship. (Option 5) Clients believe that their delusions are real despite proof otherwise. Presenting logical explanations to discredit the delusions will not help. Educational objective:When communicating with a delusional client, the nurse must focus on the client's feelings and reinforce reality rather than argue or present evidence that the delusion is false or irrational. Additional Information Psychosocial Integrity NCSBN Client Need

A client is 6 hours postoperative from hip surgery after receiving regional anesthesia and has epidural continuous anesthesia in place. Which is the most important reason for the nurse to contact the health care provider? 1. Client reports paresthesia bilaterally since the surgery (36%) 2. Fondaparinux is prescribed for STAT administration (25%) 3. Lower-extremity muscle strength is 3/5 bilaterally (9%) 4. Postoperative laboratory results show hemoglobin of 9.9 g/dL (99 g/L) (28%)

2. Fondaparinux is prescribed for STAT administration Fondaparinux (Arixtra), unfractionated heparin, and low molecular weight heparin (eg, enoxaparin, dalteparin) are anticoagulants commonly used for deep vein thrombosis and pulmonary embolism prophylaxis after hip/knee replacement or abdominal surgery. However, fondaparinux is not administered until more than 6 hours after any surgery, and anticoagulants are not given while an epidural catheter is in place (Option 2). Fondaparinux is associated with epidural hematoma. Any bleeding in the tight epidural space, which does not expand, could result in spinal cord compression. Signs of epidural spinal hematoma can include severe back pain and paralysis. (Option 1) Paresthesia is an expected finding from postoperative analgesia for 2-24 hours after surgery, depending on the agent and location. Continuously administered analgesia usually results in some paresthesia until approximately 4-6 hours after discontinuance. As long as the level remains relatively stable or improves, it is an acceptable finding. However, paresthesia or motor weakness is a concern when the sensory or motor block outlasts the expected duration. (Option 3) Client response to operative analgesia and postoperative continued analgesia can range from minimal to significant. As long as the analgesic is infusing and findings remain stable, reduced muscle strength is expected. (Option 4) Major orthopedic surgery can result in significant blood loss, and it is not unusual for the client to have hemoglobin drop of 1-2 g/dL (10-20 g/L). Blood loss should be monitored over time; transfusion usually is not indicated unless hemoglobin is <7-8 g/dL (70-80 g/L). Educational objective:Residual paresthesia and motor weakness for several hours are expected findings after regional anesthesia. Anticoagulants are not given while an epidural catheter is in place. Additional Information Pharmacological and Parenteral Therapies NCSBN Client Need

The nurse is performing a postpartum assessment 12 hours after the prolonged vaginal delivery of a term infant. Which assessment findings should be reported to the health care provider? 1. Complaints of discomfort during fundal palpation (6%) 2. Foul-smelling lochia (56%) 3. Oral temperature 100.1 F (37.8 C) (3%) 4. White blood cell (WBC) count 24,000/mm3 (24.0 x 109/L) (33%)

2. Foul-smelling lochia A foul odor of lochia suggests endometrial infection. This client has an increased risk of infection due to her prolonged labor, which involved multiple cervical examinations. The odor of lochia is usually described as "fleshy" or "musty." A foul smell warrants further evaluation. Other signs of endometrial infection are maternal fever, tachycardia, and uterine pain/tenderness. (Option 1) Palpation of the postpartum uterine fundus is commonly uncomfortable for the client. If the client complains of increasing pain, further evaluation is needed. (Option 3) Major signs and symptoms of endometrial infection include temperature above 100.4 F (38.0 C); chills; malaise; excessive uterine tenderness; and purulent, foul-smelling lochia. During the first 24 hours postpartum, the temperature is normally elevated; temperature above 100.4 F (38 C) requires further evaluation. (Option 4) The WBC count is normally elevated during the first 24 hours postpartum (up to 30,000/mm3 [30.0 x 109/L]). Leukocyte levels that are not decreasing require further evaluation. Educational objective:Signs of endometrial infection include elevated temperature, chills, malaise, excessive pain, and foul-smelling lochia. During the first 24 hours postpartum, temperature and WBC count are normally elevated. Fever and leukocyte counts that do not decrease require further evaluation. Additional Information Physiological Adaptation NCSBN Client Need

Based on the progress note documentation, which priority intervention does the nurse anticipate? Click on the exhibit button for additional information. 1. 0.9% sodium chloride, 500 mL intravenous bolus (7%) 2. Furosemide, 40 mg intravenous push (82%) 3. Metoprolol, 5 mg intravenous push (7%) 4. Vancomycin, 1 g intravenously every 12 hours (2%)

2. Furosemide, 40 mg intraveno The client's central venous pressure (CVP) is elevated (normal value 2-8 mm Hg), indicating increased systemic circulation volume and increased right ventricular preload. Pulmonary artery wedge pressure (PAWP) is also elevated (normal value 6-12 mm Hg), indicating increased left ventricular preload. In the presence of increased CVP and PAWP, coarse crackles indicate left-sided failure. The treatment goal is to decrease fluid volume and preload. Furosemide is a loop diuretic that will decrease both left- and right-sided preload. (Option 1) A fluid bolus of 500 mL of sodium chloride is contraindicated in a client with increased left and right ventricular preload as it would exacerbate fluid overload. (Option 3) Beta blockers (eg, metoprolol, atenolol, esmolol) will decrease both blood pressure and afterload. However, they will not decrease preload. (Option 4) Vancomycin is an antibiotic used to treat gram-positive bacterial infections (eg, methicillin- resistant Staphylococcus aureus); it has no effect on fluid status. Educational objective:Loop diuretics (eg, furosemide, bumetanide, torsemide) are effective in decreasing both right ventricular preload and left ventricular preload.us push (82%)

A client with Ebola was just admitted to the unit. Which actions by the nurse would represent appropriate care of this client? Select all that apply. 1. Ensuring the client wears an N95 respirator at all times 2. Keeping the door of the client's room closed at all times 3. Maintaining a log of everyone in and out of the client's room 4. Removing both pairs of gloves before removing gown and mask 5. Restricting visitors from entering the client's room

2. Keeping the door of the client's room closed at all times 3. Maintaining a log of everyone in and out of the client's room 5. Restricting visitors from entering the client's room Ebola (viral hemorrhagic fever) is an extremely contagious disease with a high mortality rate. Clients require standard, contact, droplet, and airborne precautions (eg, impermeable gown/coveralls, N95 respirator, full face shield, doubled gloves with extended cuffs, single-use boot covers, single-use apron). The client is placed in a single-client airborne isolation room with the door closed (Option 2). Visitors are prohibited unless absolutely necessary for the client's well-being (eg, parent visiting an infected child) (Option 5). For disease surveillance, a log is maintained of everyone entering or exiting the room, and all logged individuals are monitored for symptoms (Option 3). Procedures and use of sharps/needles are limited whenever possible. There are currently no medications or vaccines approved by the Food and Drug Administration to treat Ebola. Prevention is crucial. (Option 1) In a private airborne isolation room, the client does not require a respirator mask. However, all other individuals entering the room must don appropriate personal protective equipment (PPE). (Option 4) The PPE removal process after caring for a client with Ebola requires strict monitoring by a trained observer. The outer gloves are first cleaned with disinfectant and removed. The inner gloves are wiped between removal of every subsequent piece of PPE (eg, respirator, gown) and removed last. Educational objective:Ebola is an extremely contagious viral disease with a high mortality rate. Infected clients require extensive infection precautions, including an airborne isolation room, strict personal protective equipment use, restriction of visitors, and a log of individuals who enter and exit the room. Additional Information Safety and Infection Control NCSBN Client Need

The nurse is planning a client care conference with the parents of a 3-year-old with newly diagnosed type 1 diabetes mellitus. What is the priority outcome for the caregivers? 1. Demonstrating adequate coping skills (16%) 2. Knowing how to keep blood sugars stable (69%) 3. Understanding how to perform meal planning (9%) 4. Understanding the need for periodic follow-up visits (3%)

2. Knowing how to keep blood sugars stable (69% Management of type 1 diabetes mellitus requires understanding of blood sugar regulation. If the child becomes hypoglycemic or hyperglycemic, complications could develop. The priority for caregivers should be to focus on the child's safety. Managing the child's blood sugars should be the initial goal. (Option 1) Dealing with a new diagnosis will require time. Although acquiring coping skills is important, this is more of a long-term goal. (Option 3) Consistent menus, appropriate eating times, and adequate intake based on age are all important parts of meal planning. However, checking the child's blood sugars and keeping these stable with the correct insulin dose is the priority. (Option 4) Frequent follow-up visits are important to prevent long-term complications of diabetes. However, preventing hypoglycemia and hyperglycemia at home is the priority. Educational objective:Initial teaching of the parents of a child with newly diagnosed type 1 diabetes should focus on basic safety and survival skills, including proper insulin administration and adequate monitoring of blood sugars. Information should be introduced slowly, repeated often, and given based on the child's developmental age.

The nurse is caring for a newborn with patent ductus arteriosus. Which assessment finding should the nurse expect? 1. Harsh systolic murmur (14%) 2. Loud machine-like murmur (44%) 3. Soft diastolic murmur (18%) 4. Systolic ejection murmur (22%)

2. Loud machine-like murmur Patent ductus arteriosus (PDA) is an acyanotic congenital defect more common in premature infants. When fetal circulation changes to pulmonary circulation outside the womb, the ductus arteriosus should close spontaneously. This closure is caused by increased oxygenation after birth. If a PDA is present, blood will shunt from the aorta back to the pulmonary arteries via the opened ductus arteriosus. Many newborns are asymptomatic except for a loud, machine-like systolic and diastolic murmur. The PDA will be treated with surgical ligation or IV indomethacin to stimulate duct closure. (Option 1) A harsh systolic murmur is heard in the setting of ventricular septal defect, an opening between the ventricles of the heart. Ventricular septal defect is an acyanotic defect. (Option 3) A diastolic murmur is heard in mitral stenosis and aortic regurgitation but not in PDA. (Option 4) A systolic ejection murmur is heard in pulmonic stenosis. Right ventricular hypertrophy will develop if this defect is not repaired. In adults, systolic ejection murmur is usually due to aortic stenosis. Educational objective:The ductus arteriosus of a newborn should close spontaneously when fetal circulation changes to pulmonary circulation. If the ductus arteriosus remains open, blood will shunt from the aorta to the pulmonary arteries. The child will be acyanotic but will have a machine-like murmur heard on both systole and diastole.%)

The health care provider has prescribed spironolactone to be given in addition to hydrochlorothiazide to a client with hypertension. Which finding by the nurse indicates that the spironolactone is having the desired effect? 1. Blood glucose of 95 mg/dL (5.3 mmol/L) (1%) 2. Potassium level of 4.2 mEq/L (4.2 mmol/L) (79%) 3. Reduction in dizziness (7%) 4. Sodium level of 138 mEq/L (138 mmol/L) (11%)

2. Potassium level of 4.2 mEq/L (4.2 mmol/L) (79%) Potassium-sparing diuretics (eg, spironolactone, amiloride, triamterene eplerenone) are generally very weak diuretics and antihypertensives. However, they are useful when combined with thiazide diuretics to reduce potassium (K+) loss. Thiazide diuretics can cause hypokalemia when used as monotherapy. A potassium level of 4.2 mEq/L (4.2 mmol/L) falls in the normal range (3.5-5.0 mEq/L [3.5-5.0 mmol/L]), which indicates that spironolactone has been effective in preventing hypokalemia in this client receiving a thiazide diuretic (eg, hydrochlorothiazide, chlorthalidone) (Option 2). (Option 1) Blood glucose levels can be increased by thiazide diuretics but are not affected by potassium-sparing diuretics. (Option 3) All diuretics, including spironolactone, have the potential to cause dizziness. The nurse should monitor the client for orthostatic hypotension and implement safety precautions. (Option 4) Potassium-sparing diuretics exchange sodium for potassium in the kidneys; potassium is saved but sodium is lost. Therefore, a normal sodium level (135-145 mEq/L [135-145 mmol/L]) is not the desired effect. Educational objective:Potassium-sparing diuretics (eg, spironolactone, amiloride, triamterene, eplerenone) are often combined with thiazide diuretics to reduce potassium loss. Additional Information Pharmacological and Parenteral Therapies NCSBN Client Need

The health care provider (HCP) remarks that the staff nurse has a great body and that it would be worthwhile for them to have sex. The staff nurse does not want a relationship with the HCP and finds the remarks offensive. What action should the receiving nurse take initially? 1. Report the statement to the nurse manager (24%) 2. Tell the HCP to stop the comments (72%) 3. Walk away and say nothing (0%) 4. Write up an incident report (2%)

2. Tell the HCP to stop the comments (72%) Sexual harassment, including soliciting sexual favors in exchange for favorable job benefits, is prohibited. Other behaviors that could be defined as sexual harassment include asking someone for a date after the other person expressed disinterest or making remarks about a person's gender or body. The receiving nurse should first immediately and clearly indicate that the attention is unwanted and the offending HCP should stop. The offending HCP may have erroneously perceived a mutual attraction. If that is not effective, additional action should be taken. The American Nurses Association cites 4 tactics to fight workplace sexual harassment: confront, report, document, and support. (Option 1) The incident should be reported, especially if the offending HCP does not stop. If the harasser is the immediate supervisor, the receiving nurse should go up the chain of command. However, the nurse should first simply tell the offending HCP to stop and see if that resolves the issue. (Option 3) The nurse should respond with assertiveness, not avoidance. Ignoring the situation may imply that the nurse does not mind the HCP's attention. (Option 4) The receiving nurse should document what occurred and how the nurse responded. The presence of witnesses should be documented. Documentation should be stored somewhere other than the workplace. However, the nurse should initially communicate assertively that the actions are to stop before documenting them. Educational objective:A nurse who receives unwanted sexual advances in the workplace should first immediately and clearly indicate that the advances are unwanted and that the offending person should stop. Additional Information Management of Care NCSBN Client Need

A client diagnosed with septic shock has an upward-trending glucose level (180-225 mg/dL [10.0-12.5 mmol/L]) requiring control with insulin. The client's spouse asks why insulin is needed as the client is not a diabetic. What is the most appropriate response by the nurse? 1. "It is common for critically ill clients to develop type II diabetes. We give insulin to keep the glucose level under control (<140 mg/dL [7.8 mmol/L])." (3%) 2. "The client was diabetic before, but you just didn't know it. We give insulin to keep the glucose level in the normal range (70-110 mg/dL [3.9-6.1 mmol/L])." (1%) 3. "The increase in glucose is a normal response to stress by the body. We give insulin to keep the level at 140-180 mg/dL (7.8-10.0 mmol/L)." (40%) 4. "This increase is common in critically ill clients and affects their ability to fight off infection. We give insulin to keep the glucose level in the normal range (70-110 mg/dL [3.9-6.1 mmol/L])." (55%)

3. "The increase in glucose is a normal response to stress by the body. We give insulin to keep the level at 140-180 mg/dL (7.8-10.0 mmol/L)." Stress-induced hyperglycemia (gluconeogenesis) can occur in hospitalized clients in relation to surgery, trauma, acute illness, and infection. Hyperglycemia (glucose level >140 mg/dL [7.8 mmol/L]) affects both diabetic and non-diabetic hospitalized clients, especially those who are critically ill. Approximately 80% of clients in the intensive care unit who develop hyperglycemia have no history of diabetes before admission. Hyperglycemia is associated with increased risk of complications (eg, health care-associated infection, increased length of stay, acute kidney injury). To minimize complications and avoid hypoglycemia, the recommended glucose target range for critically ill clients is 140-180 mg/dL [7.8-10.0 mmol/L]. For non-critically ill clients, <140 mg/dL [7.8 mmol/L] fasting and <180 mg/dL [10.0 mmol/L] random blood glucose are recommended. (Option 1) Hospital hyperglycemia is not a direct cause of type II diabetes mellitus. In the non-diabetic client, the glucose level usually returns to normal after resolution of the disease process and/or discontinuation of steroid medications. A target glucose range of <140 mg/dL [7.8 mmol/L) is not recommended for this client. (Option 2) The prevalence of diabetes in hospitalized clients is high (about 1 in 4) and may be an undiagnosed pre-existing condition. A normal-range glucose level (70-110 mg/dL [3.9-6.1 mmol/L]) is not the recommended target range in this client due to the risk of hypoglycemia (with aggressive control) and worse outcomes. (Option 4) Although hyperglycemia does affect the ability to fight infection, 70-110 mg/dL [3.9-6.1 mmol/L] is not the recommended target range for this client. Educational objective:Stress-induced hyperglycemia causes complications in the hospitalized client. To minimize complications, the recommended target glucose range for critically ill clients is 140-180 mg/dL [7.8-10.0 mmol/L]. For non-critically ill clients, <140 mg/dL (7.8 mmol/L) fasting and <180 mg/dL (10.0 mmol/L) random blood glucose are recommended. Additional Information Physiological Adaptation NCSBN Client Need

The nurse is providing community health screening. Which of the following clients should be referred to a health care provider for further evaluation? 1. 30-year-old athlete with a heart rate of 50/min (5%) 2. 45-year-old client with a body mass index of 35 kg/m2 and fingerstick glucose of 150 mg/dL (8.3 mmol/L) (12%) 3. 55-year-old client missing all the hair on the lower legs and failing the pinprick test (71%) 4. 80-year-old client with a blood pressure of 150/90 mm Hg (10%)

3. 55-year-old client missing all the hair on the lower legs and failing the pinprick test Failure of pinprick testing indicates peripheral neuropathy. Loss of hair on the lower extremities indicates poor perfusion. The combination of these suggests peripheral neuropathy and peripheral arterial disease, likely from undiagnosed diabetes mellitus and atherosclerosis. Nearly a third of clients diagnosed with diabetes mellitus will already have complications from years of uncontrolled hyperglycemia. Diabetes mellitus dramatically accelerates the buildup of plaque on the arterial walls (atherosclerosis) when blood glucose levels are uncontrolled. (Option 1) Asymptomatic bradycardia in a healthy young adult is rarely pathological. Professional-level athletes will commonly develop athletic heart syndrome; increased efficiency results in resting sinus bradycardia (40-60/min). (Option 2) Fasting blood glucose of 150 mg/dL (8.3 mmol/L) would need to be evaluated for diabetes. However, in this case it would be important to verify whether the client has eaten recently. (Option 4) The Joint National Committee guidelines recommend against treating blood pressure readings <150/90 mm Hg in clients age >60. Educational objective:The impaired perfusion from severe atherosclerosis results in skin atrophy, poor wound healing, and widespread hair follicle death (hair loss). Additional Information Health Promotion and Maintenance NCSBN Client Need

A client diagnosed with end-stage renal disease comes to the dialysis clinic for treatment. Which actions should the nurse take to prepare the client for hemodialysis? Select all that apply. 1. Administer subcutaneous heparin to decrease clotting during dialysis 2. Administer the client's morning doses of carvedilol and lisinopril 3. Check the client's medical records to determine the last post-dialysis weight 4. Obtain a set of client vital signs and the client's current weight 5. Palpate the fistula in the client's arm for a thrill and auscultate for a bruit

3. Check the client's medical records to determine the last post-dialysis weight 4. Obtain a set of client vital signs and the client's current weight 5. Palpate the fistula in the client's arm for a thrill and auscultate for a bruit Prior to dialysis treatment, the nurse should assess the client's fluid status (weight, blood pressure, peripheral edema, lung and heart sounds), vascular access (arteriovenous fistula, arteriovenous grafts), and vital signs (Option 4). The amount of fluid removed (ultrafiltration) is determined by calculating the difference between the last post-dialysis weight and the client's current pre-dialysis weight (Option 3). After the client is connected to the dialysis machine, IV heparin is added to the blood from the client to prevent clotting that can occur when blood contacts a foreign substance. Giving subcutaneous heparin prior to initiation is not necessary (Option 1). (Option 2) During dialysis, excess fluid is removed, making the client prone to hypotension. In addition, medications are removed from the blood during hemodialysis, making them ineffective. Many medications that are taken once daily can be held until after the dialysis treatment to prevent their removal. If blood pressure medications are given prior to dialysis, the client can develop hypotension during the dialysis and then uncontrolled hypertension (decreased drug concentrations). (Option 5) Arteriovenous fistulas are created by anastomosing an artery to a vein; a thrill can be felt when palpating the fistula, and a bruit can be heard during auscultation when the fistula is functioning properly. Educational objective:The nurse is responsible for assessing the client diagnosed with end-stage renal disease for risks associated with dialysis. These risks include medication removal, hemodialysis access dysfunction, hypotension, and fluid and electrolyte imbalances.

Which client does the nurse assess first after receiving the morning report? 1. Client has cellulitis from injecting heroin; threatening to leave against medical advice if more morphine is not given right now (16%) 2. Client is 1 day postoperative colectomy; night nurse medicated client with morphine 15 minutes ago (21%) 3. Client is 2 days postoperative open gastric bypass surgery; now reporting nausea and dry heaving (61%) 4. Client is 3 days postoperative total knee replacement; waiting to be discharged (0%)

3. Client is 2 days postoperative open gastric bypass surgery; now reporting nausea and dry heaving Vomiting and dry heaving place increased mechanical stress on surgical wound edges and increase the risk for wound dehiscence and evisceration. Obese clients who have undergone extensive abdominal surgery are especially vulnerable. Therefore, the nurse should first assess the client who is nauseated and dry heaving and administer an antiemetic medication (Option 3). (Option 1) This client trying to leave against medical advice is the second priority. The nurse needs to assess this client for pain and determine when pain medication was administered last. If this situation cannot be resolved quickly, the nurse should notify the client's health care provider immediately to determine level of competency and inform the client of the risks of refusing treatment. (Option 2) The nurse must follow-up 30 minutes after the morphine is administered, not immediately, to assess the effectiveness of the pain medication. (Option 4) Providing discharge instructions to this client can wait without consequence. Educational objective:Postoperative nausea, vomiting, and dry heaving should be treated with antiemetic medication as soon as possible as it increases a client's risk for wound dehiscence and evisceration (medical emergency).

The registered nurse (RN) delegates to the unlicensed assistive personnel (UAP) the ambulation of a client. The RN observes the UAP placing the client's Foley bag on the IV pole at the level of the client's chest during the ambulation down the length of the hallway. What action should the RN take initially? 1. Discuss the need for UAP inservice education with the nurse manager (0%) 2. Give praise to the UAP for encouraging the client to walk the entire hall (0%) 3. Immediately lower the bag and speak privately to the UAP (97%) 4. Let the UAP complete assigned tasks and speak to the UAP at the end of the shift (1%)

3. Immediately lower the bag and speak privately to the UAP The Foley bag is too high and needs to be lowered. When observing a provider making an error, the RN should immediately intervene to stop any potential harm to the client. It is important to timely correct a staff member who is making a mistake to help ensure that the error is not repeated. Correction of staff should always be done privately, not in front of the client. (Option 1) Future inservice education is not a timely solution to this immediate need. It is appropriate to carry out teaching first rather than initiate disciplinary actions. According to the Federal Drug Administration's (FDA's) mandate, as no serious harm was caused, the incident does not need to be reported. (Option 2) The most important issue needing intervention is the improper positioning (too high) of the Foley catheter bag. Positive reinforcement for appropriate actions can also be included (and is beneficial), but the error should first be corrected to prevent harm. (Option 4) It is important to attend to the error right away to help ensure that the UAP does not repeat it. Letting this UAP complete assigned tasks first does not immediately deal with the incorrect position of the Foley bag and may not effectively teach (aid retention of) the correct positioning to the UAP. Educational objective:When observing a provider making an error, correct it immediately to stop any potential harm to the client. Correct the provider privately and as soon as possible. Additional Information Management of Care NCSBN Client Need

A child on the playground is experiencing an anaphylactic reaction. The school nurse arrives with an EpiPen. The weather is cold and the child is wearing several layers of clothing. How should the nurse proceed with the EpiPen? 1. Inject into the upper arm where the sleeve can be pulled up (14%) 2. Inject into the most accessible vein (1%) 3. Inject through the clothing into thigh and hold in place for 10 seconds (69%) 4. Take the child inside, remove excess clothing, and inject into the thigh (13%)

3. Inject through the clothing into thigh and hold in place for 10 seconds The EpiPen is designed to be administered through clothing with a swing and firm push against the mid-outer thigh until the injector clicks. The position should be held for 10 seconds to allow the entire contents to be injected (Option 3). The site should be massaged for an additional 10 seconds. Timing is essential in the delivery of epinephrine during an anaphylactic reaction. The nurse should administer the medication immediately on the playground without removing the child's clothing. Any delays can cause client deterioration and make maintenance of a patent airway difficult (Option 4). (Option 1) The EpiPen should be injected into the mid-outer thigh, not the upper arm. (Option 2) IV epinephrine is not administered outside the hospital setting. It requires cardiac monitoring and is indicated in clients with profound hypotension (shock) or those who do not respond to intramuscular epinephrine and fluid resuscitation. Educational objective:The EpiPen is designed to be delivered through clothing in the mid-outer thigh area. The nurse should not delay anaphylaxis treatment by attempting to remove the client's clothing.

Thrombotic thrombocytopenic purpura is suspected due to the client's current platelet count of 2,000/mm3 (2 x 109/L). Which client sign or symptom is the most concerning and requires immediate further nursing action? 1. Current oozing epistaxis (17%) 2. Ecchymosis on leg since yesterday (4%) 3. New-onset confusion (76%) 4. Reported history of hematuria (2%)

3. New-onset confusion (76%) Thrombotic thrombocytopenic purpura (TTP) consists of hemolytic anemia with fragmentation of erythrocytes, signs of intravascular hemolysis, thrombocytopenia, decreased renal function, and fever. Regardless of the cause of the low platelets, the concern in this case is the critically low (below 10,000/mm3 (10 x 109/L) platelet count, which puts this client at risk for internal bleeding, especially within the brain. Change in level of consciousness is the most clinically significant finding requiring an emergency response. (Option 1) The head is very vascular, and a nosebleed can occur with low platelets. A nosebleed is treated with direct pressure and application of cold. In this client, potential intracranial bleeding is the priority. (Option 2) Easy bruising can occur as a result of low platelets. However, the bruise is "old," and potential intracranial bleeding is the priority. (Option 4) Blood in the urine can be a symptom of low platelets due to lack of clotting ability. Although this is concerning, alterations in level of consciousness is the priority. Educational objective:A priority assessment in a client with low platelets is any change in level of consciousness (eg, disorientation, lethargy, restlessness). This can indicate intracranial bleeding and increased intracranial pressure. Additional Information Reduction of Risk Potential NCSBN Client Need

The nurse plans teaching for a client who was newly prescribed levothyroxine sodium after thyroid removal. Which instructions will the nurse include in the teaching plan? Select all that apply. 1. Drowsiness is a common side effect; taking the dose at bedtime will make this less noticeable 2. Notify the health care provider if you become pregnant as the medication is harmful to the fetus 3. Notify the health care provider if you feel a fluttering or rapid heartbeat 4. Take the medication with a meal to prevent stomach upset 5. You will need to take this medication for the rest of your life

3. Notify the health care provider if you feel a fluttering or rapid heartbeat 5. You will need to take this medication for the rest of your lif Levothyroxine sodium (eg, Levoxyl, Levothroid, Synthroid) is used to replace thyroid hormone in clients with hypothyroidism (inadequate thyroid hormone) and for those who have had their thyroid removed. These clients must understand that this medication must be taken for the rest of their lives (Option 5). A client's dose is adjusted based on serum TSH levels to prevent too much or too little hormone. Clients must be taught to report signs of excess thyroid hormone such as heart palpitations/tachycardia, weight loss, and insomnia (Option 3). (Option 1) Clients with hypothyroidism experience lethargy and somnolence. Hormone replacement therapy will increase metabolic activity and alertness. (Option 2) This medication is a hormone that is normally present in the body, so it is safe to take during pregnancy. The dose may need to be altered due to the metabolic demands of pregnancy, but the drug will not harm the fetus. (Option 4) It is best to take this medication first thing in the morning as it is best absorbed on an empty stomach (1 hour before or 2 hours after a meal). Educational objective:Clients receiving thyroid hormone replacement therapy (levothyroxine sodium) should understand that treatment is lifelong and be taught the signs of excess hormone (eg, tachycardia/palpitations, weight loss, insomnia). The medication is best absorbed on an empty stomach and is safe to take during pregnancy.e

A client with type 1 diabetes has a prescription for 20 units of NPH insulin daily at 7:30 AM and regular insulin before meals, based on a sliding scale. At 7:00 AM, the client's blood glucose level is 220 mg/dL (12.2 mmol/L), and the client's breakfast tray has arrived. What action should the nurse take? Click on the exhibit button for additional information. 1. Administer 20 units of NPH insulin now and then 6 units of regular insulin after the morning meal (3%) 2. Administer 26 units of insulin: 20 units of NPH insulin and 6 units of regular insulin in 2 separate injections (11%) 3. Administer 26 units of insulin: 20 units of NPH mixed with 6 units of regular insulin in the same syringe, drawing up the NPH into the syringe first (12%) 4. Administer 26 units of insulin: 20 units of NPH mixed with 6 units of regular insulin in the same syringe, drawing up the regular insulin first (72%)

4. Administer 26 units of insulin: 20 units of NPH mixed with 6 units of regular insulin in the same syringe, drawing up the regular insulin first Intermediate-acting insulins (NPH) can be safely mixed with short-acting (regular) and rapid-acting (lispro, aspart) insulins in one syringe (Option 4). Six units of regular insulin are needed to address the client's blood glucose reading (220 mg/dL [12.21 mmol/L]) along with the scheduled 20 units of NPH insulin. Prepare the mixed dose: Inject the NPH insulin vial with 20 units of air without inverting the vial or passing the needle into the solution. Inject 6 units of air into the regular insulin vial and withdraw the dose, leaving no air bubble. Draw NPH, totaling 26 units in one syringe. Any overdraw of NPH into the syringe will necessitate wasting the total quantity. Most long-acting insulins (eg, glargine, detemir) are not suitable for mixing and typically are packaged in prefilled injection pens. (Option 1) The 2 insulins may be safely given together before the meal because regular insulin has a rapid onset of action, whereas NPH has a slower onset but longer duration. (Option 2) The insulins can be given as 2 separate injections; however, this increases client discomfort and infection risk. (Option 3) Regular insulin should be drawn first to avoid contaminating the regular insulin vial with NPH insulin (mnemonic - RN: Regular comes before NPH). Educational objective:NPH insulin and regular insulin may be safely mixed and administered as a single injection. Regular insulin should be drawn into the syringe before intermediate-acting insulin to decrease the risk of cross-contaminating multidose vials (mnemonic - RN: Regular comes before NPH). Additional Information Pharmacological and Parenteral Therapies NCSBN Client Need

The nurse is assigned to the following clients. Which client does the nurse assess/identify as being at greatest risk for the development of a deep venous thrombosis (DVT)? 1. A 25-year-old client with abdominal pain who smokes cigarettes and takes oral contraceptives (37%) 2. A 55-year-old ambulatory client with exacerbation of chronic bronchitis and hematocrit of 56% (1%) 3. A 72-year-old client with a fever who is 2 days post coronary stent placement (7%) 4. An 80-year-old client who is 4 days postoperative from repair of a fractured hip (53%)

4. An 80-year-old client who is 4 days postoperative from repair of a fractured hip Venous thromboembolism includes both DVT and pulmonary embolism (PE). DVT is the most common form and occurs most often (80%) in the proximal deep veins (iliac, femoral) of the lower extremities. Virchow's triad describes the 3 most common theories behind the pathophysiology of the venous thrombosis: venous stasis, endothelial damage, and hypercoagulability of blood. Risk factors associated with DVT formation include the following: Trauma (endothelial injury and venous stasis from immobility) Major surgery (endothelial injury and venous stasis from immobility) Prolonged immobilization (eg, stroke, long travel) causing venous stasis Pregnancy (induced hypercoagulable state and some venous stasis by the pressure on inferior vena cava) Oral contraceptives (estrogen is thrombotic) Underlying malignancy (cancer cells release procoagulants) Smoking (produces endothelial damage by inflammation) Old age Obesity and varicose veins (venous stasis) Myeloproliferative disorders (increase blood viscosity) The 80-year-old 4-day postoperative client has the most risk factors: orthopedic hip surgery, prolonged period of immobility/inactivity, and advanced age, and is at greatest risk for developing a DVT. (Option 1) Smoking cigarettes and using oral contraceptives increase plasma fibrinogen and coagulation factors and cause hypercoagulability of blood, but the client is not at greatest risk. Hormonal contraceptives are not recommended if the client is age >35 and also smokes. (Option 2) Elevated hemoglobin/hematocrit level (erythrocytosis) causes increased blood viscosity and hypercoagulability of blood, which increases the risk for DVT. However, the client is not at greatest risk. (Option 3) Anticoagulants and antiplatelet agents are administered before and after coronary stent placement. This client is at increased risk due to endothelial damage and advanced age but is not at greatest risk. Educational objective:DVT is a frequent, often preventable complication of hospitalization, surgery, and immobilization. Factors that increase the risk for developing a DVT include trauma, surgery (especially orthopedic, knee, hip), prolonged immobility/inactivity, oral contraceptives, pregnancy, varicose veins, obesity, smoking, and advanced age. Additional Information Reduction of Risk Potential NCSBN Client Need

Based on the lung assessment information included in the hand-off report, which client should the nurse assess first? 1. Client 1-day postoperative abdominal surgery who has fine inspiratory crackles at the lung bases (19%) 2. Client with chronic bronchitis who has rhonchi in the anterior and posterior chest (5%) 3. Client with right-sided pleural effusion who has decreased breath sounds at the right lung base (30%) 4. Client with severe acute pancreatitis who has inspiratory crackles at the lung bases (45%)

4. Client with severe acute pancreatitis who has inspiratory crackles at the lung bases (45%) Clients with acute pancreatitis can develop respiratory complications including pleural effusions, atelectasis, and acute respiratory distress syndrome (ARDS). These complications are often due to activated pancreatic enzymes and cytokines that are released from the pancreas into the circulation and cause focal or systemic inflammation. ARDS is the most severe form of these complications and can rapidly progress to respiratory failure within a few hours. Therefore, the presence of inspiratory crackles in this client could indicate early ARDS and needs to be assessed further for progression. (Option 1) Fine crackles are a series of distinct, discontinuous, and high-pitched snapping sounds usually heard on inspiration. The sound originates as small atelectatic bronchioles quickly reinflate and can be expected in clients who have undergone abdominal surgery due to shallow breathing related to pain. Although the presence of fine crackles requires treatment (eg, ambulation, deep breathing), this is not the priority assessment. (Option 2) Rhonchi are continuous, low-pitched wheezes usually heard on expiration that sound like moaning or snoring. The sound originates from air moving through large airways (bronchi) filled with mucus secretions and are expected in clients with chronic bronchitis. Although they require treatment (eg, medication, mobilization of secretions), this is not the priority assessment. (Option 3) The lung under the pleural effusion is compressed, and the breath sounds are decreased/absent if auscultated over the area; this is an expected finding. Until the pleural effusion is treated with diuretics or thoracentesis, these findings will remain unchanged. Educational objective:Clients with acute pancreatitis are at high risk for developing acute respiratory distress syndrome. Additional Information Management of Care NCSBN Client Need

A client is brought to the emergency department after sustaining third-degree burns over 50% of the body. Which solution is the best choice for fluid resuscitation in this client? 1. 0.45% normal saline (7%) 2. 5% dextrose in 0.9% normal saline (D5NS) (12%) 3. 5% dextrose in water (D5W) (2%) 4. Lactated Ringer's solution (77%)

4. Lactated Ringer's solution (77%) The greatest immediate threat to a client with severe and extensive burn injuries is hypovolemic shock and electrolyte imbalance. This is due to cellular damage and increased capillary permeability caused by direct thermal trauma, which result in fluid loss. In the emergent phase of burn management, it is critical to establish an airway and replenish lost intravascular fluid, proteins, and electrolytes. Lactated Ringer's (LR), also known as Ringer's lactate, is the solution of choice for fluid resuscitation of a burned client due to its similarity in chemical composition to human plasma (Option 4). LR remains in the intravascular space longer than other solutions, which helps to stabilize blood pressure and avert shock. (Option 1) Hypotonic solutions (eg, 0.45% normal saline) quickly leave the intravascular space and are not useful in replacing intravascular volume. They may also contribute to peripheral and interstitial edema, which can lead to pulmonary complications. (Option 2) Hypertonic solutions (eg, 5% dextrose in 0.9% normal saline [D5NS], 3% saline) can cause further electrolyte imbalances in a client with severe burns, resulting in hypernatremia, hyperchloremia, and arrhythmias. (Option 3) Although technically an isotonic solution, 5% dextrose in water (D5W) behaves as a hypotonic solution when dextrose is metabolized by the body and free water is released to the tissues rather than remaining in the intravascular space. Educational objective:Lactated Ringer's is the standard for fluid resuscitation in burn clients due to its similarity in chemical composition to human plasma. Hypotonic, hypertonic, and dextrose-containing solutions should not be used for fluid resuscitation.

A nurse is caring for a client on a mechanical ventilator. The ventilator is sounding an alarm and displaying an alert about low tidal volumes. The nurse has checked all connections and the endotracheal tube, but the alarm persists and the client's oxygen saturation is dropping. What should the nurse do next? 1. Call the respiratory therapist to the bedside to troubleshoot (3%) 2. Elevate the head of the bed and apply a nonrebreather mask (18%) 3. Increase the oxygen delivery on the ventilator to 100% (8%) 4. Manually ventilate with a resuscitation bag device attached to the endotracheal tube (68%)

4. Manually ventilate with a resuscitation bag device attached to the endotracheal tube A low tidal volume alarm indicates that the volume of air the ventilator is delivering is lower than the set volume. This is most often due to a disconnection, loose connection, or leak in the circuit. The nurse should troubleshoot the most common causes of the alarm, but if the client's condition is deteriorating clinically (eg, decreasing oxygen saturation), then the nurse should disconnect the ventilator and manually ventilate the client's lungs with a resuscitation bag device at 10-15 L/min oxygen until the ventilator alarm state can be resolved. (Option 1) Respiratory therapists have specialized training in mechanical ventilators. They should be called to the bedside but only after the nurse has begun to stabilize the client's condition using manual ventilation. (Option 2) Since the client is intubated, air cannot pass from the nares and oropharynx into the lungs, and ventilation can be achieved only via the endotracheal tube. (Option 3) The client would benefit from a higher oxygen level, but the ventilator is unable to deliver the programmed volume to the client, even with an increased oxygen level. The client's lungs must be manually ventilated with a resuscitation bag. Educational objective:Ventilators may sound an alarm when set parameters are not being met (eg, low tidal volumes, high peak pressures). These alarms may indicate a client condition or ventilator malfunction. If a ventilator alarm cannot be readily resolved, the nurse should manually ventilate the client's lungs with a resuscitation bag device. Additional Information Physiological Adaptation NCSBN Client Need

The nurse is supervising a graduate nurse (GN) on a telemetry unit. An assigned client develops asystole with no pulse, and emergency care interventions are initiated. Which action by the GN would cause the supervising nurse to intervene? 1. Administers IV epinephrine (22%) 2. Applies oxygen with bag-mask (17%) 3. Initiates chest compressions (3%) 4. Provides defibrillator shock (56%)

4. Provides defibrillator shock The client in asystole has a total absence of ventricular electrical activity and is pulseless, apneic, and unresponsive. The nurse should first verify the monitor reading by assessing the client and palpating for a pulse, and then call for help and initiate emergency care (ie, CPR, oxygenated ventilation). Defibrillation is not indicated when there is no electrical activity present (ie, asystole) or when the heart muscle is not contracting despite an organized rhythm (ie, pulseless electrical activity [PEA]). Defibrillation attempts to convert lethal ventricular dysrhythmias (ie, ventricular fibrillation and pulseless ventricular tachycardia) into an organized rhythm by passing an electric shock through the heart. Defibrillation cannot create an organized rhythm if there is no electrical activity in the heart (Option 4). (Options 1, 2, and 3) Immediate interventions for asystole and PEA include CPR and oxygenated ventilation. Advanced cardiovascular life-support measures include epinephrine IV, placement of advanced airway (ie, intubation), and treatment of reversible causes (eg, hypovolemia, hyperkalemia). When treating asystole or PEA, the absolute priority is providing continuous, high-quality CPR and oxygenated ventilation until circulation spontaneously returns or the client enters into a shockable rhythm. Unfortunately, restoration of circulation may not be possible, and clients in asystole often cannot be resuscitated. Educational objective:Asystole is characterized by a total absence of ventricular electrical activity. The client is pulseless, apneic, and unresponsive. Treatment includes CPR, oxygenated ventilation, and advanced cardiovascular life-support measures (eg, epinephrine IV, advanced airway). Defibrillation is not effective for treatment of asystole or pulseless electrical activity.

The nurse working in an extended care facility transcribes a prescription from the health care provider for a single daily dose of 150 mg of ranitidine; this is to be taken orally at bedtime for treatment of gastroesophageal reflux disease. Of the following prescriptions, which one is transcribed correctly? 1. Ranitidine 150 mcg daily by mouth (2%) 2. Ranitidine 150 mg per os qhs (3%) 3. Ranitidine 150 mcg po qd nightly (2%) 4. Ranitidine 150 mg PO at bedtime (91%)

4. Ranitidine 150 mg PO at bedtime The nurse has correctly transcribed the prescription using approved abbreviations and standard terminology. (Option 1) The abbreviation mcg signifies microgram. This option is incorrect as the medication is to be given daily in units of milligrams and it must be taken specifically at bedtime. (Option 2) The abbreviation per os is interpreted as by mouth. The United States Pharmacopeia Institute for Safe Medication Practices Medication Error Reporting Program recommends that per os not be used as it may be read mistakenly as left eye. It equally recommends that the abbreviation qhs not be used as it may be misinterpreted as qhr or every hour, leading to a potential medication administration error. (Option 3) The abbreviation mcg signifies microgram, making this option incorrect. The abbreviation qd is on the do-not-use list of The Joint Commission National Patient Safety Goals; it may be mistakenly read as qid (four times daily), which may cause a serious medication administration error. Educational objective:Using approved abbreviations when transcribing health care provider prescriptions promotes client safety and prevents potential medication administration errors. Common abbreviations (per os, qhs, qd) can result in errors and should not be used. Additional Information Management of Care NCSBN Client Need

A 1-year-old child who goes to day care is recovering from an episode of otitis media. Which intervention is most important for the nurse to recommend to the parents in order to prevent recurrence? 1. Exclusive breastfeeding (2%) 2. Not sending the child to day care (1%) 3. Preventing water from entering the ear (44%) 4. Smoking cessation by the parents (51%)

4. Smoking cessation by the parents (51%) Otitis media (OM) is the inflammation or infection of the middle ear resulting from dysfunction of the eustachian tube. OM typically occurs in infants and children under age 2, sometimes following a respiratory tract infection. The eustachian tubes in infants and young children are short, straight, and fairly horizontal, which results in ineffective drainage and protection from respiratory secretions. Infants with exposure to tobacco smoke are at risk for OM due to the resulting respiratory inflammation. OM risk is also higher with activities such as using a pacifier or drinking from a bottle when lying down as these allow fluid to pool in the mouth and then reach the eustachian tubes. Key preventive measures include eliminating exposure to smoke, obtaining routine immunizations to prevent infection, and reducing or eliminating use of a pacifier after age 6 months. (Option 1) Breast-fed infants have a decreased risk for OM, possibly due to the semivertical position used when breastfeeding, which reduces reflux to the eustachian tubes. Exclusive breastfeeding is recommended for the first 6 months. However, this client is age 1 and should be receiving a varied, healthy intake of solid food at this time. (Option 2) Day care attendance is a significant risk factor to the development of OM. However, the recommendation to avoid day care is usually not practical as many parents must work outside of the home. (Option 3) Excess water in the ears from bathing or swimming can alter the protective environment of the external ear and contribute to otitis externa, known as swimmer's ear; however, this does not contribute to OM. Educational objective:Otitis media, inflammation of the middle ear, commonly occurs in children under age 2. Key interventions for prevention include avoiding exposure to tobacco smoke, obtaining routine immunizations, and discontinuing use of a pacifier after age 6 months. Additional Information Health Promotion and Maintenance NCSBN Client Need

During the shift report, the night charge nurse tells the day charge nurse that the night unlicensed assistive personnel (UAP) is totally incompetent. What is the best response for the day charge nurse to give? 1. Encourage the night nurse to provide the UAP with additional training (33%) 2. Indicate that it is the night nurse's job to deal with staff problems (1%) 3. Remind the night nurse that the UAP is doing the best job the UAP can (1%) 4. Suggest that the night nurse discuss concerns with the nurse manager (64%)

4. Suggest that the night nurse discuss concerns with the nurse manager Incompetency is a concern for client safety and quality care. The nurse manager is responsible for hiring/firing and setting up additional training times or experiences for staff. The situation should be discussed with the person who has 24/7 responsibility for the unit so that an appropriate response can be given to the night nurse's perceptions (Option 4). (Option 1) The night nurse can provide task-specific instructions/training, but incompetence implies a global dysfunction beyond minor, on-the-job, intermittent instructions. In addition, other factors could be involved that may be influencing the UAP's behavior, such as personal issues or impairment from substance abuse. It is best to discuss this situation with a higher authority to determine the best approach. (Option 2) This response is something the night nurse knows. The need is to decide the next action. The scope of this problem is probably beyond the night nurse's responsibility and authority. (Option 3) This response may be true. However, the bottom line is finding out if the UAP's performance is of adequate quality and safe for clients. The amount of effort that the caregiver is expending is not the bottom line. Educational objective:When a caregiver's performance is below the standard of care needed to provide safe and quality care to clients, the appropriate authority should be notified so that the situation can be handled.


संबंधित स्टडी सेट्स

Chapter 1: Strategic Management and Strategic Competitiveness

View Set

Geometry Chapter 5 - Congruent Triangles

View Set

From Gutenberg to Google- Test 8

View Set

Abeka World Geography Test 6 Final Exam!

View Set

Banking: How to Manage Your Money

View Set